SurgiNotes 2022 - Questions 2010-2021
SurgiNotes 2022 - Questions 2010-2021
SurgiNotes 2022 - Questions 2010-2021
[Questions 2010-2021]
بسم هللا الرحمن الرحيم
الملف ده فيه امتحانات خاصه بالجزء األول للزمالة المرصية ف الجراحة العامة و تخصصتا.
عباره عن :3 Parts
Part A
• الجزء ده عباره عنه 12امتحان ( MCQوىه كل االمتحانات المتاحة من 2010ل :)2021
1. 2021 September - Paper (1) MCQ [Not Answered].
2. 2021 March - Paper (2) MCQ.
3. 2020 October - Paper (2) MCQ.
4. 2020 October - Paper (1) MCQ.
5. 2019 July - Paper (1) MCQ.
6. 2018 July - Paper (1) MCQ.
7. 2018 February - Paper (1) MCQ.
8. 2017 February - Paper (1) MCQ.
9. 2015 March - Paper (1) MCQ.
10.2014 March - Paper (1) MCQ.
11.2013 June - Paper (1) MCQ.
12.2010 January - Paper (1) MCQ.
• االمتحانات دي كلها مجابه من فايل د O.A. /ما عدا االمتحان االول لسه اجاباته مش متوفرة
سني اىل فاتت بنسبه
بس الحمد هلل االمتحان لحسن الحظ متكرر منه كت ري من أسئلة ال ر
%60تقريبا.
• فيه بعض األسئلة لألسف مش موجوده ف الورق األصل اىل ناقل منه االمتحانات وده ف
امتحاني بس الحمد هلل:
ر
1. 2021 September - Paper (1) MCQ:
a. 13 – 14 – 15 – 16 – 17.
2. 2018 July - Paper (1) MCQ:
a. 4 – 3 – 2 – 1.
b. 13 – 12 – 11 – 10 – 9.
c. 44 – 43 – 42 – 41 – 40.
d. 97 – 96 – 95 – 94 – 93.
• برده فيه بعض الكلمات ماكنتش واضحة او مش موجوده في بعض األسئلة بس الحمد هلل مش
كتير:
1. 2020 October - Paper (2) MCQ: 65.
2. 2018 July - Paper (1) MCQ: 90.
Part B
• الجزء ده عباره عنه 7امتحانات ( Writtenوىه كل االمتحانات المتاحة من 2010ل
:)2021
1. 2019 July - Paper (2) Written.
2. 2018 July - Paper (2) Written.
3. 2018 February - Paper (2) Written.
4. 2017 February - Paper (2) Written.
5. 2013 June - Paper (2) Written.
6. 2013 February - Paper (2) Written.
7. 2012 June - Paper (2) Written.
• فيه بعض الكلمات ماكنتش واضحة او مش موجوده في بعض األسئلة بس الحمد هلل في امتحان
واحد:
1. 2013 June - Paper (2) Written: 9 a – c – d.
Part C
• الجزء ده عباره عنه 3أجزاء MCQف ال ( Basicمتكرر فيها أسئلة ر
كتي ف االمتحانات
المتاحة من 2010ل :)2021
1. Surgical Anatomy MCQ.
2. Antibiotics Pharmacology MCQ.
3. Physiology MCQ.
. • فيه بعض األسئلة متكررة ف نفس الفايل سبتها لزياده التأكيد للناس اىل بتنس برسعه
غي
ف النهاية انا جمعت االمتحانات واألسئلة دي بغرض ان احنا نساعد بعض ومش محتاج منكم ر
دعوه بظهر الغيب ولو حد ليه أي تعديل عل اجابه او أي حد قدر يوصل ل األسئلة او الكلمات اىل
ناقصه ما رييددش انه يقول عليها ......
ELEMAM’s SurgiNotes
ElEMAM’s SurgiNotes 2022
5. The principal blood supply to the parathyroid glands is which of the following?
a. Superior thyroid arteries.
b. Inferior thyroid arteries.
c. Thyroid ima arteries.
d. Parathyroid arterial branches directly from the external carotid artery.
e. Vertebral artery.
6. A 73-year-old lady suffers a fracture at the surgical neck of the humerus. During
surgical correction, which of the following vessels lie posterior to the fracture?
a. Axillary artery.
b. Brachial artery.
c. Thoracoacromial artery.
d. Transverse scapular artery.
e. Posterior circumflex humeral artery.
7. The femoral canal is a point of weakness in the abdominal wall. Which of the
following statements is the MOST accurate regarding the anatomy of the
femoral canal?
a. The femoral artery forms the anterior border of the femoral canal.
b. The femoral vein forms the lateral border.
c. The inguinal ligament lies posterior to the canal.
d. The pectineal part of the inguinal ligament is anterior to the canal.
e. The pectineal ligament forms the medial border.
P a g e 4 | 25
ElEMAM’s SurgiNotes 2022
19.Pain referred to the right side of the neck and extending laterally from the right
clavicle to the tip of the right shoulder is MOST likely due to involvement of:
a. Cervical cardiac nerves.
b. Posterior vagal trunk.
c. Right intercostal nerves.
d. Right phrenic nerve.
e. Right recurrent laryngeal nerve.
P a g e 5 | 25
ElEMAM’s SurgiNotes 2022
22.A patient who had an injury to the right lateral side of the neck was unable to
raise his right shoulder. What is the injured nerve?
a. Accessory.
b. Dorsal scapular.
c. Greater occipital.
d. Thoracodorsal.
e. Transverse cutaneous nerve of the neck.
24.A 30-year-old male patient was subjected to a motor car accident and he had
only a fracture of the upper end of the fibula. At examination the patient had
sensory loss at the dorsum of the right foot. Which else may be found on clinical
examination of the patient?
a. Sensory loss at the middle side of the foot.
b. Sensory loss at the sole of the foot.
c. Weak dorsiflexion of the foot.
d. Weakness in flexion of the knee.
e. Weakness in planter flexion of the foot.
P a g e 6 | 25
ElEMAM’s SurgiNotes 2022
25.A 20-year-old man presented with a stab injury to the anterior aspect of the
right wrist. On examination he was found to have a deep 2 cm long transverse
laceration at the front of the right wrist with loss of sensation in the ring and
little fingers. The patient was unable to abduct and adduct his fingers on the
affected side. Radial pulse was palpable. What was the MOST likely structure to
have been injured?
a. Anterior interosseous nerve.
b. Median nerve.
c. Musculocutaneous nerve.
d. Radial nerve.
e. Ulnar nerve.
26.A 30-year-old male had a stab wound at the front of the forearm. Examination
revealed loss of flexion of the distal pharynx of the thumb and the lateral 3
fingers. Which of the following was the injured nerve?
a. Ulnar nerve.
b. Posterior interosseous nerve.
c. Anterior interosseous nerve.
d. Radial nerve.
e. Musculocutaneous nerve.
28.A CT scan of a 63-year-old man reveals that the left renal vein is occluded as it
crosses the aorta. The occlusion is due to compression by an arterial aneurysm
anterior to the vein. What is the MOST likely location of this aneurysm?
a. Coeliac artery.
b. Inferior mesenteric artery.
c. Left colic artery.
d. Middle colic artery.
e. Superior mesenteric artery.
P a g e 7 | 25
ElEMAM’s SurgiNotes 2022
29.A 24 year old female received a small-caliber bullet wound to the popliteal fossa.
The surgeons recognized that the bullet had severed the tibial nerve. Such an
injury would MOST likely result in which of the following?
a. Inability to extend the leg at the knee.
b. Foot drop.
c. A dorsiflexed and everted foot.
d. A plantar flexed and inverted foot.
e. Total inability to flex the leg at the knee joint.
31.A 25-year-old man is playing football when he complains of posterior thigh pain
whilst sprinting with the ball. On examination he has tenderness in the lower
lateral posterior thigh. He is unable to fully extend the knee due to pain. What
is the MOST likely injury?
a. Biceps femoris tear.
b. Gracilis tear.
c. Gastrocnemius tear.
d. Semimembranosus tear.
e. Semitendinosus tear.
P a g e 8 | 25
ElEMAM’s SurgiNotes 2022
33.A 65-year-old man presents with haematuria and right loin pain. Computerized
tomography demonstrates a right renal tumor and he undergoes a right radical
nephrectomy. The right testicular vein drains into which of the following?
a. Inferior mesenteric vein.
b. Inferior vena cava.
c. Right adrenal vein.
d. Right lumbar vein.
e. Right renal vein.
34.A 58-year-old male farmer was accidentally injured by a sharp tool. During
physical examination the doctor noted that the patient has foot drop, although
sensation was present over the dorsum of the foot and the skin of the posterior
calf. Which of the following nerve was injured?
a. Femoral nerve.
b. Sciatic nerve.
c. Superficial fibular (Peroneal) nerve.
d. Deep fibular (Peroneal) nerve.
e. Common fibular (Peroneal) nerve.
35.Which of the following cell types is essential for normal wound healing?
a. Erythrocytes.
b. Leukocytes.
c. Lymphocytes.
d. Monocytes
e. Platelets.
38.The cervical esophagus receives its blood supply primarily from the:
a. Internal carotid artery.
b. Inferior thyroid artery.
c. Superior thyroid artery.
d. Inferior cervical artery.
e. Facial artery.
39.Which of the following is the MOST potent stimulus for ADH hormone
secretion?
a. Decreased plasma osmolarity.
b. Decreased plasma volume.
c. Hypothalamic releasing factor.
d. Increased plasma osmolarity.
e. Increased plasma volume.
40.Which of the following increases the tone of the lower esophageal sphincter?
a. Gastrin.
b. Secretin.
c. Cholecystokinin.
d. Caffeine.
e. Calcium channel blocker.
P a g e 10 | 25
ElEMAM’s SurgiNotes 2022
41.A patient, who presented with severe diarrhea as a result of clostridium difficile,
develops a metabolic acidosis. This is MOST likely due to loss of which one of
the following ions in the diarrhea?
a. Cl-.
b. HCO3-.
c. K+.
d. NH4+.
e. ОН-.
43.Primary malignant tumors of which of the following endocrine glands does NOT
present with hyperfunction?
a. Thyroid.
b. Parathyroid.
c. Endocrine pancreas.
d. Adrenal medulla.
e. Adrenal cortex.
44.A 26-year-old patient presents with abdominal pain, weight loss and
steatorrhea. He is found to have severe terminal ileal involvement. Which of the
following vitamins is the LEAST likely to be deficient?
a. Vitamin A.
b. Vitamin D.
c. Vitamin E.
d. Vitamin C.
e. Vitamin K.
P a g e 11 | 25
ElEMAM’s SurgiNotes 2022
49.In the presence of acute inflammation, when an abscess has formed, which of
the following cell types is MOST likely to be primarily involved in phagocytic
activity?
a. Erythrocytes.
b. Lymphocytes.
c. Macrophages.
d. Mast cells.
e. Plasma cells.
51.Which of the following is the MOST effective way in preventing surgical site
infection?
a. Antibiotic prophylaxis.
b. Bowel preparation.
c. Drains and irrigation.
d. Skin preparation.
e. Surgical technique.
53.The medial mammary artery supplying the medial aspect of the breast is a
tributary of the:
a. 2nd, 3rd and 4th intercostal arteries.
b. Internal mammary artery.
c. Thoracoacromial artery.
d. Posterior intercostal arteries.
e. Musculo-phrenic artery.
54.Which of the following is TRUE regarding the physiology of the thyroid gland?
a. Absorption of iodides occurs by passive diffusion.
b. Potassium perchlorate prevents the union between iodine and tyrosine.
c. T3 is four times more active than T4.
d. The half-life of thyroxine is 4 days.
e. The necessary intake of iodide is 50 ug/day.
65.Which of the following is the MOST common cause of death in patients with
ovarian cancer?
a. Uremia.
b. Anemia.
c. Liver failure.
d. Bowel obstruction.
e. Respiratory failure.
67.A 56-year-old lady presents with a pathological bone fracture, which of the
following is the likely primary source?
a. Thyroid.
b. Breast.
c. Kidney.
d. Endometrium.
e. Colon.
68.Central venous access lines are a minor procedure, but may associated with
many complications, the COMMONEST is:
a. Haematoma.
b. Air embolism.
c. Pneumothorax.
d. Arrhythmias.
e. Lost wire or catheter.
P a g e 17 | 25
ElEMAM’s SurgiNotes 2022
70.Which of the following is the BEST parameter for monitoring septic shock?
a. Central venous pressure (CVP).
b. Vasopressor requirement.
c. Urine output.
d. Serum lactate.
e. Mental status changes.
71.A 55-year-old male patient is receiving 150 mg of aspirin daily because he has
coronary artery disease. Which of the following tests will be affected in this
patient?
a. Bleeding time.
b. Coagulation time.
c. INR.
d. PTT.
e. Thrombin time.
72.A 40-year-old male patient had a car accident which led to fracture pelvis. The
patient received 4 litres of stored blood following which he had evidence of
generalized coagulopathy. Deficiency of which of the following factors is the
cause of the problem?
a. Fibrinogen.
b. II.
c. V, VIII.
d. XI.
e. XII.
P a g e 18 | 25
ElEMAM’s SurgiNotes 2022
75.What does increased level of fibrin degradation products (FDP) in the blood
denote?
a. Diminished fibrinogen synthesis.
b. Diminished platelet count.
c. Diminished platelet function.
d. Disseminated intravascular coagulation (DIC).
e. Heparin overdose.
77.A 22-year-old man is brought into the emergency department in profound shock
after a fall from the fourth floor of a building. After resuscitation, small bowel
resection and hepatic segmentectomy are performed at laparotomy. He
receives 15 unit of packed RBCs, 4 unit of fresh-frozen plasma and 8 L of Ringer’s
lactate. On closure, diffuse oozing of blood is noted. What is the MOST likely
cause?
a. Hepatic failure.
b. Hypersplenism.
c. Platelet deficiency.
d. Factor IX (Christmas factor) deficiency.
e. Congenital hypoprothrombinemia.
79.Which of the following types of shock is associated with high pulmonary wedge
pressure?
a. Hypovolemic shock.
b. Cardiogenic shock.
c. Early septic shock.
d. Late septic shock.
e. Neurogenic shock.
85.A 40 years old dark-skinned male had an excision of a parotid tumor that
resulted in an ugly scar. Which information applies to his condition?
a. Keloids contain an overabundance of fibroblasts.
b. A hypertrophic scar extends beyond the boundaries of the original
wound.
c. Improvement is usually seen with keloid excision followed by intralesional
steroid injection.
d. An incision placed perpendicular to the lines of natural skin tension will
result in the least obvious scar.
e. Hypertrophic scars occur most commonly on the lower extremities.
86.A 78-year-old woman with emphysema on oxygen mask has the following blood
gas results:
pH PO2 PaCO2 Bicarbonate Base excess
Finding: 7.28 70 mm Hg 48 mm Hg 36 mmol/L +5
What is the MOST likely interpretation?
a. Mixed respiratory and metabolic acidosis.
b. Partially compensated metabolic acidosis.
c. Partially compensated respiratory acidosis.
d. Uncompensated metabolic acidosis.
e. Uncompensated respiratory acidosis.
95.The primary lymphatic drainage of the midline of the upper lip is:
a. Submandibular nodes.
b. Submental nodes.
c. Intraparotid nodes.
d. Preauricular nodes.
e. Jugulo-omohyoid nodes.
96.More than 80% of accessory spleens are found in the splenic hilum. What is the
second MOST common location for an accessory spleen?
a. The Lienorenal ligament.
b. The greater omentum.
c. The gastrocolic ligament.
d. The tail of the pancreas.
e. The splenocolic ligament.
98. Which of the following is the primary function of the spleen in human adults?
a. Production of red cells.
b. Production of white cells.
c. Destruction of platelets.
d. Storage of blood.
e. Host defense.
100. The MOST common position of the right recurrent laryngeal nerve is:
a. Anterior to the inferior thyroid artery.
b. Posterior to the inferior thyroid artery.
c. Between the branches of the inferior thyroid artery.
d. Transversely parallel to inferior thyroid artery.
e. Absent (Nonrecurrent) laryngeal nerve.
P a g e 25 | 25
ElEMAM’s SurgiNotes 2022
3. What is the hormone that acts on the intestines to increase calcium absorption?
a. Calcitonin.
b. Corticotrophin releasing factor (CRF).
c. Pancreatic polypeptide.
d. Parathormone.
e. Thyroxine.
5. Regarding the anatomy of the thyroid gland, which of the following TRUE?
a. In about 80% of persons, the recurrent laryngeal nerve traverses anterior
to the inferior thyroid artery.
b. The recurrent laryngeal nerve has an oblique course around the
subclavian artery on the left side.
c. The superior laryngeal nerve provides both sensory and motor function
to the larynx.
d. The thyroid gland is innervated only by parasympathetic fibers from the
vagus nerve.
e. Unilateral recurrent laryngeal nerve injury usually results in airway
compromise that necessitates tracheotomy.
8. You have a patient who has pheochromocytoma and his blood pressure is
200/120 mmHg. You are preparing this patient for surgery. Which of the
following drugs should be used FIRST for the control of blood pressure?
a. Phenoxybenzamine.
b. Propranolol.
c. Nifedipine.
d. Atenolol.
e. Captopril.
P a g e 2 | 25
ElEMAM’s SurgiNotes 2022
9. A 45-yearold female is found to have a 2-cm solid nodule in her right adrenal
gland at the time of an abdominal CT scan following an auto-accident. With
regard to the adrenal lesion, the patient is asymptomatic and the nodule is
found to be non-functional on evaluation. What is the recommended
management?
a. Extraperitoneal right adrenalectomy through either a flank of posterior
approach.
b. Suppression with 5 mg prednisone PO t.d.s.
c. Follow-up CT scan after 3 months.
d. Excision biopsy via laparoscopic approach.
e. Arterial embolization.
12.A 50-year-old woman presents with lethargy, weight gain, cold intolerance and
loss of interest for the past six months. Which is the MOST appropriate initial
investigation?
a. Erythrocyte sedimentation rate estimation.
b. Radioactive isotope scan of thyroid.
c. Thyroid antibodies screen.
d. Thyroid stimulating hormone estimation.
e. Ultrasound scan of thyroid gland.
P a g e 3 | 25
ElEMAM’s SurgiNotes 2022
13.A patient with chronic renal failure attends the emergency department
complaining of increasing confusion, muscle weakness, nausea, vomiting and
fatigue. The serum calcium level is 12.4 mg/dl. The FIRST step in management
of this patient should be:
a. Emergency parathyroidectomy.
b. Aggressive intravenous hydration.
c. Initiation of furosemide infusion.
d. Continuous calcitonin infusion.
e. Initiation of bisphosphonates.
16.A 42-year-old woman is in the intensive care unit immediately following removal
of a left adrenal pheochromocytoma. Her blood pressure is 80/40 mmHg. The
MOST appropriate treatment of the patient's hypotension is:
a. Epinephrine.
b. IV bolus of lactated Ringer solution.
c. Methylprednisolone.
d. Phenoxybenzamine.
e. Phenylephrine.
P a g e 4 | 25
ElEMAM’s SurgiNotes 2022
24.On workup, a patient is found to have elevated free cortisol and plasma ACTH
levels. Further testing reveals that both low- and high-dose dexamethasone
administration fail to suppress cortisol production. What is the MOST likely
diagnosis?
a. Bilateral adrenal hyperplasia.
b. Pituitary tumor.
c. Adrenal adenoma.
d. Ectopic ACTH-producing tumor.
e. Exogenous corticosteroids.
P a g e 6 | 25
ElEMAM’s SurgiNotes 2022
P a g e 7 | 25
ElEMAM’s SurgiNotes 2022
P a g e 9 | 25
ElEMAM’s SurgiNotes 2022
37.A 24-year-old woman has acute renal failure following postpartum hemorrhage.
Laboratory studies showed serum glucose 150 mg/dl; sodium 135 mEq/L;
potassium 6.5 mEq/L; chloride 105 mEq/L and bicarbonate 15 mEq/L. Which of
the following is recommended?
a. Decrease potassium chloride to 10 mEq/L.
b. Intravenous 0.9% sodium chloride.
c. 100 ml of 50% glucose water with 10 U insulin.
d. Intravenous calcitonin.
e. Intravenous magnesium sulfate.
38.Following an operation and extubation, a patient was restless. His arterial blood
gases revealed: pH 7.36; PaO2 65 mmHg; PaCO2 55 mmHg; PaCO3- 38 mmol/L.
The physiologic status can BEST be described as which of the following?
a. Respiratory alkalosis.
b. Respiratory acidosis.
c. Metabolic acidosis.
d. Metabolic alkalosis.
e. Combined respiratory and metabolic acidosis.
40.A primary tumor of which of these organs is the LEAST likely to metastasize to
bone:
a. Breast.
b. Colon.
c. Kidney.
d. Lung.
e. Prostate.
P a g e 10 | 25
ElEMAM’s SurgiNotes 2022
41.A patient, who had gastrectomy 10 days ago developed massive pulmonary
embolism proved by CT angiography. What is the recommended treatment?
a. IV heparin.
b. Fibrinolytic agents.
c. Warfarin.
d. IV acetylsalicylic acid.
e. Corticosteroids.
43.A 39-year-old woman is making a slow but adequate recovery after sustaining a
40% surface area burn injury. On the sixth postoperative day she becomes
unwell. She vomits intermittently, has painless abdominal distension and starts
to hiccup. What is the MOST likely cause of these symptoms?
a. Acute gastric dilatation.
b. Acute intestinal obstruction.
c. Clostridium difficile infection.
d. Fecal impaction.
e. Systemic sepsis.
44.The arterial blood gases of a patient who had an operation for ruptured aortic
aneurysm are: pH 7.54, PO2 100 mmHg, PCO2 30 mmHg, HCO3- 30 mmol/L. What
is the acid-base disturbance?
a. Respiratory acidosis.
b. Respiratory alkalosis.
c. Metabolic alkalosis.
d. Combined respiratory and metabolic alkalosis.
e. Compensated respiratory acidosis.
P a g e 11 | 25
ElEMAM’s SurgiNotes 2022
45.A 78-year-old man with history of coronary artery disease requests an elective
epigastric hernia repair. Which of the following is valid reason for delaying the
proposed surgery?
a. Coronary artery bypass surgery 3 months earlier.
b. A history of cigarette smoking.
c. Jugular venous distension.
d. Hypertension.
e. Hyperlipidemia.
47.Which of the following types of shock is associated with high pulmonary wedge
pressure?
a. Hypovolemic shock.
b. Cardiogenic shock.
c. Early septic shock .
d. Late septic shock.
e. Neurogenic shock.
51.A 38-year-old man in end-stage renal failure resulting from polycystic kidney
disease receives a cadaveric renal transplant. Good renal function is established
but four weeks later deteriorates, the serum creatinine rising by 25%. Which of
the following processes is MOST likely to be responsible for this deterioration?
a. B-cell mediated rejection.
b. Circulating immune complex disease.
c. IgG antibody mediated rejection.
d. Post-transplant lymphoproliferative disorder.
e. T-cell mediated rejection.
52.A 78-year-old woman with emphysema receiving 28% oxygen by mask has the
following blood gas results:
pH PO2 PaCO2 Bicarbonate Base excess
Finding: 7.28 70 mm Hg 48 mm Hg 36 mmol/L +5
Normal: 7.35-7.45 90-110 35-45 22-26 -2 to +2
The MOST likely interpretation is:
a. Mixed respiratory and metabolic acidosis.
b. Partially compensated metabolic acidosis.
c. Partially compensated respiratory acidosis.
d. Uncompensated metabolic acidosis.
e. Uncompensated respiratory acidosis.
P a g e 13 | 25
ElEMAM’s SurgiNotes 2022
P a g e 14 | 25
ElEMAM’s SurgiNotes 2022
57.A 55-year-old male patient has been receiving TPN for prolonged time and
developed hypomagnesaemia. Which of the following conditions clinically
resembles hypomagnesaemia?
a. Hypoglycemia.
b. Hypokalemia.
c. Hypophosphatemia.
d. Hypocalcemia.
e. Hyponatremia.
59.A 21-year-ald man undergoes major abdominal surgery after a motor vehicle
collision. He has a cardiac arrest in the intensive care unit shortly after returning
from surgery. Select the MOST appropriate pharmacologic agent for the patient:
a. Epinephrine.
b. Norepinephrine.
c. Phenylephrine.
d. Dopamine.
e. Dobutamine.
60.Which of the following is the BEST parameter for monitoring septic shock?
a. Central venous pressure (CVP).
b. Vasopressor requirement.
c. Urine output.
d. Serum lactate.
e. Mental status changes.
P a g e 15 | 25
ElEMAM’s SurgiNotes 2022
64.The arterial blood gas results of a 40-year-old male patient who had an accident
one week ago are: pH 7.54; PO2 80 mmHg; PCO2 30 mmHg; PCO3- 18 mmol/L.
The MOST likely cause is:
a. Excessive vomiting.
b. Pulmonary embolism.
c. Diabetic ketoacidosis.
d. CVA.
e. Persistent diarrhea.
P a g e 16 | 25
ElEMAM’s SurgiNotes 2022
66.Which of the following options is MOST appropriate for a type II diabetic patient
undergoing bilateral hernia repair as a day-case procedure under general
anesthesia?
a. Their usual diabetic medication should be omitted on the night before
surgery.
b. They should be prescribed an insulin/dextrose sliding scale starting at 6
a.m. on the day of surgery.
c. They should be placed first on the operating list whenever possible.
d. They should be kept nil by mouth at least 6 hours before the procedure.
e. They should receive long acting insulin at the night of the operation.
68.A 70-year-old man undergoes anterior resection for carcinoma of the rectum.
He is extubated in the operating room (OR). In the recovery room, he is found
to be restless with an HR of 136 bpm and a BP of 144/80 mmHg. ABG analysis
on room air reveals pH 7.24; PCO2 60 mmHg; PO2 54 mmHg; HCO3- 25 mEq/L
and SaO2 90%. Appropriate management for this patient should be which of the
following?
a. To administer 40% oxygen by mask.
b. Morphine, 2 mg IV.
c. C. Ringer's lactate 250 mL over 1 hour.
d. Intubation and ventilatory support.
e. Deep breathing and coughing.
P a g e 17 | 25
ElEMAM’s SurgiNotes 2022
69. Clinical studies have shown that administration of lactated Ringer's solution to
patients with hypovolemic shock may:
a. A Increase serum lactate concentration.
b. Impair liver function.
c. Improve hemodynamics by alleviating the deficit in the interstitial fluid
compartment.
d. It increases metabolic acidosis.
e. Increase the need for blood transfusion.
70.Central venous access lines is a minor procedure, but may associated with many
complications, the COMMONEST is:
a. Haematoma.
b. Air embolism.
c. Pneumothorax.
d. Arrhythmias.
e. Lost wire or catheter.
71. The gold standard for diagnosing pulmonary embolism (PE) is:
a. CT pulmonary angiogram.
b. MRI.
c. Ventilation perfusion nuclear scan (VQ scan).
d. Duplex ultrasound.
e. ECG.
72.What is the MOST common tumor that metastasizes to the adrenal gland?
a. Breast cancer.
b. Bronchogenic cancer.
c. Melanoma.
d. Renal cell carcinoma.
e. Thyroid cancer.
P a g e 18 | 25
ElEMAM’s SurgiNotes 2022
75.A 40-year-old man presents to his doctor with a swelling in the neck of eight
weeks' duration. Clinical examination demonstrates that he has multiple
swellings in the neck, axillae and both inguinal regions. What is the MOST likely
diagnosis?
a. Acute lymphadenitis.
b. Cat-scratch disease.
c. Malignant lymphoma.
d. Metastatic carcinoma.
e. Reactive Lymphadenopathy.
82.A 56-year-old lady presents with a pathological fracture of the proximal femur.
Which of the following primary sites is the MOST likely source of her disease?
a. Thyroid.
b. Breast.
c. Kidney.
d. Endometrium.
e. Colon.
83.Which of these option is the COMMON cause of death in patients with ovarian
cancer?
a. Uremia.
b. Anemia.
c. Liver failure.
d. Bowel obstruction.
e. Respiratory failure.
90.The chest X-ray of a 68-year-old intubated man in the surgical intensive care unit
reveals an infiltrate in his right lower lobe. Concurrent with this finding, the
patient has an elevated leukocyte count and increased endotracheal secretions.
Which of the following procedures would have been MOST effective in
preventing ventilator-associated pneumonia (VAP)?
a. Elevation of the head of bed above 30 degrees.
b. Initiation of ranitidine.
c. Replacement of nasogastric tube with nasoduodenal tube for post pyloric
enteral feeding.
d. Selective decontamination of the digestive tract.
e. Use of prophylactic topical antibiotics (Intratracheal or oral).
92.A 35-year-old male patient was subjected to a car accident and had multiple rib
fractures and fracture of the right shaft femur. The pulse is 120/min and the BP
is 90/60 mmHg. The patient is in severe agony. What is the MOST appropriate
parenteral analgesic for this patient?
a. Morphine.
b. Fentanyl.
c. Midazolam.
d. Paracetamol.
e. Propofol.
P a g e 23 | 25
ElEMAM’s SurgiNotes 2022
93.Which site or venue is the area that is at GREATEST risk for surgical errors?
a. Operating room.
b. Surgical intensive care unit.
c. Hospital wards/floors.
d. Emergency department.
e. Ambulatory care sites.
94.An unconscious accident victim is hypotensive from intra-abdominal
hemorrhage and needs an emergency laparotomy. His identity is unknown and
therefore, no family is available. Which of the following should be done?
a. Nothing, it is illegal to operate on a patient without consent.
b. The surgeon should document the need for the surgery in the chart and
proceed.
c. Three doctors should document the need for the surgery in the chart and
the surgeon should then proceed.
d. A court order for surgery should be obtained prior to proceeding.
e. Inform local health authority and then proceed.
95.Surgical "Never Events" are errors in medical care that are clearly identifiable,
preventable and serious in their consequences for patients and that indicate a
real problem in the safety and credibility of a health care facility. Which surgical
error does NOT belong to this category?
a. Surgery performed on the wrong body part.
b. Surgery performed on the wrong patient.
c. Immediately postoperative death in an ASA class II patient.
d. Wrong surgical procedure performed on a patient.
e. A foreign body left in a patient after surgery.
96.Before skin incision "Time Out" is a phase of the surgical safety check list. What
is the CORRECT step that should be done at this phase?
a. The Checklist coordinator will verbally review with the patient his
identity.
b. The Checklist coordinator will verbally review with the patient that the
procedure and site are correct.
c. The Checklist coordinator will verbally review with the patient that
consent for surgery has been given.
d. The team will confirm that prophylactic antibiotics have been
administered within the previous 60 minutes.
e. The team will review together the operation that was performed,
completion of sponge and instrument counts.
P a g e 24 | 25
ElEMAM’s SurgiNotes 2022
2. A 40-year-old male patient had excision of a carotid body tumor. Which of the
following nerves is MOST liable to be injured?
a. Facial nerve.
b. Glossopharyngeal nerve.
c. Hypoglossal nerve.
d. Spinal accessory nerve.
e. Vagus nerve.
3. Intracapsular fractures of the neck of the femur are very liable to be followed
by avascular necrosis of the head of femur. From which arteries does the head
of femur receive its MAIN blood supply?
a. Artery of the ligament of the head of femur.
b. Deep external pudendal artery.
c. First perforator artery of the deep femoral.
d. Medial and lateral circumflex femoral arteries.
e. The superficial femoral artery.
4. Taste from the posterior one third of tongue is provided by which of the
following nerves?
a. Facial.
b. Glossopharyngeal.
c. Hypoglossal.
d. Lingual.
e. Vagus.
P a g e 1 | 25
ElEMAM’s SurgiNotes 2022
5. Pain referred to the right side of the neck and extending laterally from the right
clavicle to the tip of the right shoulder is MOST likely due to involvement of:
a. Cervical cardiac nerves.
b. Posterior vagal trunk.
c. Right intercostal nerves.
d. Right phrenic nerve.
e. Right recurrent laryngeal nerve.
7. Which of the following will be present following injury of the long thoracic
nerve?
a. Depression of the shoulder.
b. Inability to abduct the shoulder.
c. Inability to extend the shoulder.
d. Loss of sensation at the medial side of the axilla.
e. Winging of the scapula.
9. Which of the following is TRUE regarding the anatomy of the thyroid gland?
a. In about 80 % of persons, the recurrent laryngeal nerve traverses anterior
to the inferior thyroid artery.
b. The recurrent laryngeal nerve has an oblique course around the
subclavian artery on the left side.
c. The superior laryngeal nerve provides both sensory and motor function
to the larynx.
d. The thyroid gland is innervated only by parasympathetic fibers from the
vagus nerve.
e. Unilateral recurrent laryngeal nerve injury usually results in airway
compromise that necessitates tracheotomy.
10.A patient who had an injury to the right lateral side of the neck was unable to
raise his right shoulder. What is the injured nerve?
a. Accessory.
b. Dorsal scapular.
c. Greater occipital.
d. Thoracodorsal.
e. Transverse cutaneous nerve of the neck.
12.A patient has severe hematemesis due to bleeding ulcer on the lesser curvature
of the stomach. During surgery which artery needs to be ligated to stop the
bleeding?
a. Gastroduodenal.
b. Left gastric.
c. Left gastroepiploic.
d. Right gastroepiploic.
e. Short gastric.
P a g e 3 | 25
ElEMAM’s SurgiNotes 2022
14. A 30-year-old male patient was subjected to a motor car accident and he had only
a fracture of the upper end of the fibula. At examination the patient had sensory
loss at the dorsum of the right foot. Which else may be found on clinical
examination of the patient?
a. Sensory loss at the middle side of the foot.
b. Sensory loss at the stole of the foot.
c. Weakness in dorsiflexion of the foot.
d. Weakness in flexion of the knee.
e. Weakness in planter flexion of the foot.
15. A 25-year-old male patient is involved in a serious car accident. Examination reveals
that his right leg is shortened and internally rotated. The patient is unable to
dorsiflex or planter flex his foot. All sensations below the knee are lost apart from
the medial side of the leg and foot and upper back of the calf. Which of the
following is the MOST likely injured nerve?
a. Common peroneal nerve.
b. Tibial nerve.
c. Obturator nerve.
d. Sciatic nerve.
e. Femoral nerve.
16. A 20-year-old man presented with a stab injury to the anterior aspect of the right
wrist. On examination he was found to have a deep 2 cm long transverse laceration
at the front of the right wrist with loss of sensation in the ring and little fingers. The
patient was unable to abduct and adduct his fingers on the affected side. Radial
pulse was palpable. What was the MOST likely structure to have been injured?
a. Anterior interosseous nerve.
b. Median nerve.
c. Musculocutaneous nerve.
d. Radial nerve.
e. Ulnar nerve.
P a g e 4 | 25
ElEMAM’s SurgiNotes 2022
17.A 19-year-old woman attends the emergency department after being shot in
the neck with an air-gun pellet. In the course of surgical exploration of the
posterior triangle, a nerve is injured. The patient is unable to shrug her left
shoulder or fully abduct her left arm. Which is the MOST likely structure to have
been injured?
a. Axillary nerve.
b. Dorsal scapular nerve.
c. Long thoracic nerve.
d. Spinal accessory nerve.
e. Suprascapular nerve.
18.Which of the following statement is TRUE regarding surgical excision of the right
submandibular salivary gland?
a. Injury to the lingual nerve would result in loss of sensations to the
posterior one third of the right side of the tongue.
b. Injury to the hypoglossal nerve would result in deviation of the tongue to
the right side on protrusion of the tongue.
c. The great auricular nerve is at a risk of injury.
d. Injury of the marginal mandibular branch of the facial nerve would result
in sensory loss at the angle of the mandible.
e. Injury to the cervical branch of the facial nerve would result in drooping
of the right angle of the mouth.
19.A 30-year-old male had a stab wound of the front of the forearm. Examination
revealed loss of flexion of the distal pharynx of the thumb and the lateral 3
fingers. Which of the following was the injured nerve?
a. Ulnar nerve.
b. Posterior interosseous nerve.
c. Anterior interosseous nerve.
d. Radial nerve.
e. Musculocutaneous nerve.
20.A 24-year-old woman presents to the outpatient clinic with pelvic pain. A CT
scan reveals enlarged para-aortic lymph nodes. These are MOST likely to be
involved in secondary spread from a tumor in which of the following organs?
a. Cervix.
b. Ovary.
c. Uterus.
d. Vagina.
e. Vulva.
P a g e 5 | 25
ElEMAM’s SurgiNotes 2022
22.A 30-year-old motorcyclist suffers a closed fracture to the mid-shaft of the tibia.
Anterior compartment syndrome could cause loss of sensation:
a. In the first web space.
b. Over the dorsum of the foot.
c. Over the lateral edge of the foot.
d. Over the medial aspect of the hallux.
e. Over the medial malleolus.
P a g e 6 | 25
ElEMAM’s SurgiNotes 2022
25.A CT scan of a 63-year-old man reveals that the left renal vein is occluded as it
crosses the aorta. The occlusion is due to compression by an arterial aneurysm
anterior to the vein. What is the MOST likely location of this aneurysm?
a. Coeliac artery.
b. Inferior mesenteric artery.
c. Left colic artery.
d. Middle colic artery.
e. Superior mesenteric artery.
26.A 27-year-old man is admitted to the emergency department after a car crash.
Physical examination reveals weakness in medial rotation and adduction of the
humerus. Which of the following nerves was MOST probably injured?
a. Thoracodorsal.
b. Axillary.
c. Dorsal scapular.
d. Spinal accessory.
e. Radial.
27.A 19-year-old man is brought to the emergency department after dislocating his
shoulder while playing football. Following treatment of the dislocation, he
cannot initiate abduction of his arm. An MRI of the shoulder shows a torn
muscle. Which muscle was MOST likely damaged by the injury?
a. Coracobrachialis.
b. Long head of the triceps.
c. Pectoralis minor.
d. Supraspinatus.
e. Teres major.
29.A 60-year-old-male male accidentally injured his wrist with a knife, partially
dividing the ulnar nerve. Which of the following actions would MOST likely be
lost as a result of this injury?
a. Flexion of the proximal interphalangeal joint of the fifth digit (Little
finger).
b. Extension of the thumb.
c. Adduction of the fifth digit.
d. Abduction of the thumb.
e. Opposition of the thumb.
30.A 43-year-old woman cannot hold a piece of paper between her thumb and the
lateral side of her index finger without flexing the distal joint of her thumb.
Weakness of which specific muscle causes this sign to appear?
a. Flexor pollicis longus.
b. Adductor pollicis.
c. Flexor digiti minimi.
d. Flexor carpi radialis.
e. Extensor indicis.
33.Which of the following cell types is essential for normal wound healing?
a. Erythrocytes.
b. Leukocytes.
c. Lymphocytes.
d. Monocytes.
e. Platelets.
34.A 50-year-old male patient is suffering from septic shock due to leaking
intestinal anastomosis. Which of the following is the BEST index for the severity
of shock?
a. Creatinine level.
b. Hb %.
c. Lactate level.
d. O₂ saturation of arterial blood.
e. Plasma sodium concentration.
35.Which of the following is TRUE regarding the physiology of the thyroid gland?
a. Absorption of iodides occurs by passive diffusion.
b. Potassium perchlorate prevents the union between iodine and tyrosine.
c. T3 is four times more active than T4.
d. The half-life of thyroxine is 4 days.
e. The necessary intake of iodide is 50 ug/day.
38.A 60-year-old male patient is suffering from diabetic ketoacidosis. Which of the
following is the MOST important buffer base in the extracellular fluid?
a. Bicarbonate.
b. Hemoglobin.
c. Lactate.
d. Phosphate.
e. Plasma proteins.
39.The following acid-base data: pH: 7.2, PCO₂: 20 mmHg, HCO3-: 8 mmol/L, base
excess: -19 mmol/L, would be MOST consistent with:
a. Anxiety.
b. Lobar collapse of the lung.
c. Pyloric obstruction.
d. Septic shock.
e. Starvation.
43.A 25-year-old man presents to the emergency department after being involved
in a road traffic accident. He is conscious, talking, has a blood pressure of 90/70
mmHg and a heart rate of 100 beats/minute. He has sustained a fractured pelvis
and femur. Which of the following is TRUE?
a. His total peripheral resistance is decreased.
b. The discharge rate of his carotid sinus nerves is increased.
c. There is a decrease in renin production.
d. There is an increase in angiotensin II.
e. There is an increase in renal sodium excretion.
45.A 78-year-old woman with emphysema receiving 28% oxygen by mask has the
following blood gas results:
pH PO2 PaCO2 Bicarbonate Base excess
Finding: 7.28 70 mm Hg 48 mm Hg 36 mmol/L +5
What is the MOST likely interpretation?
a. Mixed respiratory and metabolic acidosis.
b. Partially compensated metabolic acidosis.
c. Partially compensated respiratory acidosis.
d. Uncompensated metabolic acidosis.
e. Uncompensated respiratory acidosis.
52.A 40-year-old male patient was injured in the thigh and a bleeding point was
ligated by a silk suture. The patient got recurrent attacks of inflammation at the
site of the wound. The surgeon explored the wound and removed an
inflammatory mass around the silk suture. Which of the following cells will be
the predominant cell detected in this mass?
a. Basophils.
b. Eosinophils.
c. Lymphocyte.
d. Monocytes.
e. Neutrophils.
P a g e 13 | 25
ElEMAM’s SurgiNotes 2022
54.In patients with reflux esophagitis. What does the presence of columnar cells in
the esophageal mucosa represent?
a. Carcinoma in situ.
b. Carcinoma.
c. Dysplasia.
d. Hyperplasia.
e. Metaplasia.
57.A 55-year-old male patient is receiving 150 mg of aspirin daily because he has
coronary artery disease. Which of the following tests will be affected in this
patient?
a. Bleeding time.
b. Coagulation time.
c. INR.
d. PTT.
e. Thrombin time.
58.A 30-year-old male patient was admitted to the casualty department due to a
car accident. The patient had fracture of the pelvis and the right femur and he
received 5 liters of blood following which he started to have bleeding from is
nose and mouth. What is the MAIN cause of this bleeding tendency?
a. Decrease in fibrinogen.
b. Decrease in prothrombin.
c. Decrease of calcium.
d. Increased fibrinolytic activity.
e. Platelet depletion.
59.A patient with a life threatening pulmonary embolus is receiving heparin. She
developed serious vaginal bleeding and a major retroperitoneal haematoma
after 5 days of heparin therapy. What is the recommended treatment?
a. Reverse heparin and evacuate the haematoma.
b. Reverse heparin by protamine sulphate and insert a vena caval filter.
c. Stop heparin and closely observe the patient.
d. Stop heparin, give fresh frozen plasma and start warfarin therapy.
e. Switch to low-dose heparin.
60.Which of the following is the MOST likely cause of increased prothrombin time?
a. Christmas disease.
b. Hemophilia.
c. Heparin therapy.
d. Von Willebrand’s disease.
e. Warfarin overdose.
P a g e 15 | 25
ElEMAM’s SurgiNotes 2022
64.What does increased level of fibrin degradation products (FDP) in the blood
denote?
a. Diminished fibrinogen synthesis.
b. Diminished platelet count.
c. Diminished platelet function.
d. Disseminated intravascular coagulation (DIC).
e. Heparin overdose.
P a g e 16 | 25
ElEMAM’s SurgiNotes 2022
67.A 5-year-old boy slipped and hurt his right knee while walking. He presents with
a tender, swollen, warm knee with significant hemarthrosis. His PT is 12
(Normal: 13 seconds), PTT is over 100 (Normal: 25 seconds), platelet count is
300,000/mm³ and bleeding time is normal. Initial management should consist
of which of the following?
a. Fresh-frozen plasma.
b. Aspiration of knee.
c. Factor VIII concentrate.
d. Passive exercise.
e. Long-leg cast.
69.A 50-year-old female patient has chronic renal failure and has been maintained
an chronic dialysis. The patient underwent cholecystectomy. Post-operatively
she had severe bleeding. What is the MOST likely cause for this bleeding?
a. Elevated PT.
b. Elevated PTT.
c. Low platelet count.
d. Decreased platelet aggregation.
e. Sepsis.
70.A 70-year-old female patient is receiving warfarin because she has AF and had
previous thrombosis. Which of the following statements regarding warfarin is
CORRECT?
a. The dose of warfarin is adjusted according to the partial thromboplastin
time.
b. Warfarin takes about 8 hours to exert its effects.
c. Warfarin acts by inhibiting factor XII.
d. If the patient is also taking aspirin, the dose of warfarin should be
reduced.
e. Protamine sulphate is the antidote to warfarin.
74.Trauma patients sustaining what type of injury are at HIGHEST risk of venous
thromboembolism?
a. Head trauma.
b. Femur fracture.
c. Pelvic fracture.
d. Splenectomy.
e. Spinal cord injury.
75.Which of the following is the MOST effective way in preventing surgical site
infection?
a. Antibiotic prophylaxis.
b. Bowel preparation.
c. Drains and irrigation.
d. Skin preparation.
e. Surgical technique.
77. A 45-year-old diabetic male patient complains of severe pain in the thigh.
Examination reveals spreading oedema, swelling and tenderness of the thigh. A
provisional diagnosis of necrotizing fasciitis is made. Which of the following is
MOST important in the treatment?
a. Antifungal agents.
b. Antitoxin.
c. Hyperbaric O₂.
d. Immunoglobulins.
e. Wide surgical debridement.
78. A patient with a non-obstructing carcinoma of the sigmoid colon being prepared
for elective resection. Which of the following reduces the risk of postoperative
infectious complications?
a. Avoidance of oral antibiotics to prevent emergence of Clostridium difficile.
b. Postoperative administration for 48 h of parenteral antibiotics effective
against aerobes and anaerobes.
c. Postoperative administration of parenteral antibiotics effective against
aerobes and anaerobes until the patient's intravenous lines and all other
drains are removed.
d. Redosing of antibiotics in the operating room if the case lasts for more than
2 hours of operating time.
e. Single preoperative parenteral dose of antibiotic effective against aerobes.
82.A 30-year-old woman sustained a puncture wound to the foot. The patient has
been on a therapeutic dose of steroids for the past 5 years for ulcerative colitis.
Her last tetanus toxoid booster was 8 years ago. What should the patient
receive?
a. Tetanus toxoid booster.
b. Human immunoglobulin, antibiotics with anaerobic coverage.
c. Tetanus toxoid plus human immunoglobulin.
d. Tetanus toxoid plus human immunoglobulin and antibiotics with aerobic
and anaerobic coverage.
e. Wide debridement of the wound.
P a g e 23 | 25
ElEMAM’s SurgiNotes 2022
94.Postoperative wound infections after operations for head and neck cancer are
MOST frequently caused by:
a. Bacillus fragilis.
b. Escherichia coli.
c. Gram negative anaerobes.
d. Staphylococcus aureus.
e. Pseudomonas aeruginosa.
100. The MOST common source of bacteria in wound infection after groin hernia
repair is:
a. The patient skin.
b. The patient nasopharynx.
c. Operating room air.
d. Surgical instruments.
e. Gastrointestinal flora.
P a g e 25 | 25
ElEMAM’s SurgiNotes 2022
P a g e 1 | 25
ElEMAM’s SurgiNotes 2022
9. Following an operation and extubation, a patient was restless. His arterial blood
gases revealed: pH 7.36; PaO2 65 mmHg; PaCO2 55 mmHg; HCO3- 36 mmol/L.
The physiologic status can BEST be described as which of the following?
a. Respiratory alkalosis.
b. Respiratory acidosis.
c. Metabolic acidosis.
d. Metabolic alkalosis.
e. Combined respiratory and metabolic acidosis.
P a g e 3 | 25
ElEMAM’s SurgiNotes 2022
14.A 65 year old man is noted to have a blood pressure of 90/62 mmHg on the
evening after an uncomplicated small bowel resection for obstruction. His heart
rate is 110/mm, respiratory rate 24/mm and temperature 37.4 oC, urine output
is only 20 ml over 2 hours and oxygen saturation by pulse oximetry is 95%. His
pre-operative hemoglobin was 12.6 g/dl. Which of the following statements is
the MOST accurate regarding the patient?
a. Initial therapy should be intravenous crystalloid fluid bolus.
b. Intravenous furosemide (Lasix) should be administered.
c. This patient is most likely affected by anxiety and a mild anxiolytic and
careful observation should be initiated.
d. A hemoglobin level performed in the recovery room after surgery of 12.4
g/dl is good evidence against active hemorrhage.
e. Urgent CT abdomen is required.
15.A patient, who had gastrectomy 10 days ago developed massive pulmonary
embolism proved by CT angiography. What is the recommended treatment?
a. IV heparin.
b. Fibrinolytic agent.
c. Warfarin.
d. IV acetylsalicylic acid.
e. Corticosteroids.
17.A 70-year-old man with chronic obstructive pulmonary disease is admitted for
elective hemicolectomy. What is a preoperative arterial blood gas analysis likely
to show?
Arterial PCO2: Bicarbonate:
a. Decreased. Decreased.
b. Decreased. Increased.
c. Decreased. Normal.
d. Increased. Decreased.
e. Increased. Increased.
18.A 39-year-old woman making a slow but adequate recovery after sustaining a
40% surface area burn injury. on the sixth postoperative day she becomes
unwell. She vomits intermittently, has painless abdominal distension and starts
to hiccup. What is the MOST likely causes of these symptoms?
a. Acute gastric dilatation.
b. Acute intestinal obstruction.
c. Clostridium difficile infection.
d. Fecal impaction.
e. Systemic sepsis.
20.Which of the following types of shock is associated with high pulmonary wodge
pressure?
a. Hypovolemic shock.
b. Cardiogenic shock.
c. Early septic shock.
d. Late septic shock.
e. Neurogenic shock.
P a g e 5 | 25
ElEMAM’s SurgiNotes 2022
24.The anion gap will increase with an increase in the plasma concentration of:
a. Sodium.
b. Potassium.
c. Chloride.
d. Bicarbonate.
e. Lactate.
P a g e 6 | 25
ElEMAM’s SurgiNotes 2022
25. A 78-year-old woman with emphysema receiving 28% oxygen by mask has the
following blood gas results:
pH PO2 PaCO2 Bicarbonate Base excess
Finding: 7.28 70 mm Hg 48 mm Hg 36 mmol/L +5
Normal: 7.35-7.45 90-110 35-45 22-26 -2 to +2
The MOST likely interpretation is:
a. Mixed respiratory and metabolic acidosis.
b. Partially compensated metabolic acidosis.
c. Partially compensated respiratory acidosis.
d. Uncompensated metabolic acidosis.
e. Uncompensated respiratory acidosis.
26. A 28-year-old motorcyclist is admitted following a road traffic accident, having
sustained bilateral femoral fractures and a ruptured spleen. Three days
postoperatively he is noted to be agitated, hypoxaemic and difficult to ventilation
with a BP of 120/80 mmHg, regular pulse of 88 a beats/minute and he is apyrexial.
A chest X-ray shows bilateral diffuse lung infiltrates. What is the MOST likely
underlying diagnosis?
a. Adult respiratory distress syndrome (ARDS).
b. Atelectasis.
c. Bronchopneumonia.
d. Pulmonary oedema.
e. Pulmonary thrombo-embolism.
27. A 22-year-old man involved in a motor vehicle accident is found to have a thoracic
spine fracture (T6) and paraplegia. The patient is hypotensive with a systolic BP of
70 mmHg, is bradycardiac with a pulse of 48 beats/min and is breathing
comfortably. Which of the following would the MOST appropriate initial
treatment?
a. Isotonic fluid administration.
b. Steroid administration within 24 hours of the injury.
c. Immediate intubation.
d. Alpha-agonist administration.
e. Immediate magnetic resonance imaging.
28. Which of the following is TRUE regarding nosocomial pneumonia among intensive
care unit patients?
a. Has the same mortality rate as does community-acquired pneumonia.
b. Is the most common nosocomial infection.
c. Can be avoided by early tracheostomy.
d. Is directly related to the duration of intubation.
e. Can be prevented by early institution of prophylactic antibiotics.
P a g e 7 | 25
ElEMAM’s SurgiNotes 2022
30.A 55-year-old male patient has been receiving TPN for prolonged time and
developed hypomagnesaemia which of the following conditions clinically
resembles hypomagnesaemia?
a. Hypoglycemia.
b. Hypokalemia.
c. Hypophosphatemia.
d. Hypocalcemia.
e. Hyponatremia.
33.Which of the following is the BEST parameter for monitoring septic shock?
a. Central venous pressure (CVP).
b. Vasopressor requirement.
c. Urine Output.
d. Serum lactate.
e. Mental status changes.
37.A 35-year-old woman presents to you after running her first marathon with
complaints of muscle aches. Which of the following is NOT an action of cortisol
in this metabolically stressed patient?
a. It stimulates release of insulin by the pancreas.
b. It induces insulin resistance in muscles and adipose tissue.
f. It stimulates release of lactate from skeletal muscle.
c. It induces release of glycerol from adipose tissue.
d. It leads to immunosuppression.
39.Which of the following statements relating to large volume blood loss in trauma
is INCORRECT?
a. Tranexamic acid reduces the incidence of rebleeding following surgery.
b. Hypocalcaemia may complicate resuscitation.
c. Colloids are preferred initially as they reduce the incidence of
coagulopathy.
d. When patients receive over 5 units of whole blood, mortality increases
when blood products greater than 3 weeks old are utilized.
e. In the battlefield setting a ratio of one unit fresh blood to one unit plasma
is often utilized.
41.A 45-year-old male patient had renal transplantation operation. Thirty minutes
after the operation the urinary output markedly dropped. The blood pressure
of was 120/80 mmHg. Duplex scanning of the transplanted kidney revealed
patent renal vessels and normal pelvicalyceal system. Which of the following is
TRUE regarding the present problem?
a. It is less liable to occur in females who had repeated pregnancies.
b. It is more liable to occur in patients who receive liver transplantation.
c. Stimulated CD4 lymphocytes are mainly responsible for the problem.
d. The patient needs immediate removal of the transplanted kidney.
e. The problem can be corrected by increasing the dose of cyclosporine.
43.A 40-year-old male patient underwent a renal transplant com a cadaveric donor.
Immediately after finishing the vascular anastomosis, the kidneys became
cyanotic and flaccid. Histological examination revealed deposition of
immunoglobulins and complement in the vessel walls. What is the
immunological background of the problem?
a. Donor cytotoxic T-lymphocytes directed against the front antigens.
b. Donor natural killer cells directed against host antigens.
c. Host natural killer cells against donor antigens.
d. Preformed donor antibodies against the host antigens.
e. Preformed host antibodies against the donor antigens.
47.One week after receiving a cadaver renal allograft, the recipient remains oliguric
and dialysis dependent. Ultrasonography reveals a large perigraft fluid
collection. Appropriate next step in the management of this patient is:
a. Observation.
b. Aspiration of the perigraft fluid and installation of fibrosis inducing agent
to obliterate the dead space.
c. Aspiration of the perigraft fluid collection for chemical analysis.
d. CT of the abdomen.
e. Angiography for localization of the bleeding site.
48.A 37-year-old woman with end-stage renal disease had a cadaveric renal
transplant 18 months previously. She tolerated her transplantation well and has
been receiving a stable regiment of tacrolimus and corticosteroids with a
creatinine of 1.0 mg/dl. She now presents with fatigue, bone pain and
osteoporosis. Her serum calcium is 11.5 mg/dl. The next step in management
should be:
a. Subtotal parathyroidectomy.
b. An increase in corticosteroid dose.
c. Hydration and diuresis.
d. Calcitonin injections.
e. Phosphate binders.
P a g e 12 | 25
ElEMAM’s SurgiNotes 2022
57. Which of the following statements regarding adrenal cortical insufficiency is TRUE?
a. Treatment with exogenous steroids is usually ineffective.
b. It is commonly seen as a consequence of metastasis of distant tumors, such
as lungs or breast, to the adrenal glands.
c. Chronic adrenal insufficiency (Addison's disease) in the preoperative patient
should be recognized by constellation of findings, including the
hyperglycemia, hypernatremia and hypokalemia.
d. Death from untreated chronic adrenal insufficiency may occur within hours
of surgery.
e. The most common underlying cause today is infection with resistant
microorganisms.
58. A 55-year-old female patient who has metastatic breast cancer presents with
weakness, anorexia, malaise, constipation and back pain and lethargy. Laboratory
studies include a normal chest X-ray; serum albumin 3.2 mg/dl; serum calcium 14
mg/dl; serum phosphorus 2.6 mg/dl; serum chloride 108 mg/dl; BUN 32 mg/dl and
creatinine 2.0 mg/dl. What is the appropriate initial management?
a. Intravenous normal saline infusion.
b. Administration of thiazide diuretics.
c. Administration of intravenous phosphorus.
d. Use of mithramycin.
e. Neck exploration and parathyroidectomy.
59. You have a patient who has pheochromocytoma and his blood pressure 200/120
mmHg. You are preparing this patient for surgery. Which of the following drugs
should be used FIRST for the control of blood pressure?
a. Phenoxybenzamine.
b. Propranolol.
c. Nifedipine
d. Atenolol.
e. Captopril.
60. After undergoing a thyroidectomy operation. 42-year-old opera singer has no
change in speech, but she has difficulty in singing high-pitched voices. Which nerve
is MOST likely to be injured?
a. Recurrent laryngeal.
b. Internal laryngeal.
c. External laryngeal.
d. Pharyngeal branch of vagus.
e. Phrenic.
P a g e 15 | 25
ElEMAM’s SurgiNotes 2022
61.50-year-old woman presents with lethargy, weight gain, cold intolerance and
loss of interest for the past six months. Which is the MOST appropriate initial
investigation?
a. Erythrocyte sedimentation rate estimation.
b. Radioactive isotope scan of thyroid.
c. Thyroid antibodies screen.
d. Thyroid stimulating hormone estimation.
e. Ultrasound scan of thyroid gland.
62.40-year-old man is admitted to the surgical day case unit for repair of his left
inguinal hernia. On examination he is noted to have diffuse skin tanning, spotty
pigmentation of the elbows, nipples and buttocks and pigmentation of the scar
from a previous right inguinal hernia repair. Three hours after the operation he
becomes severely hypotensive. What is the MOST likely cause?
a. ACTH deficiency.
b. Adrenal insufficiency.
c. Growth hormone deficiency.
d. Potassium deficiency.
e. Thyroxin deficiency.
63.A pregnant mother in her first trimester comes to her clinician office with a
diagnosis of primary hyperparathyroidism. What is the CORRECT management?
a. Parathyroidectomy during the second trimester.
b. Parathyroidectomy during the third trimester.
c. Prescribing a calcimimetic agent to help reduce hypercalcemia until after
delivery, when definitive surgery can be offered safely.
d. Close observation and parathyroidectomy following delivery.
e. Weekly injections of calcitonin until delivery, when definitive surgery can
be offered safely.
64.A patient who had adrenalectomy to remove a pheochromocytoma
demonstrates signs of confusion and complains of sweating and headache
several hours following his operation. His blood pressure is 130/65 mmHg, his
heart rate is 100 beats/min and his respiratory rate is 12 breaths/min. What is
the MOST likely cause of his symptoms?
a. Dehydration.
b. Postoperative bleeding.
c. Hypoglycemia.
d. Narcotic overdose.
e. Incomplete removal of the pheochromocytoma.
P a g e 16 | 25
ElEMAM’s SurgiNotes 2022
68.A 42-year-old woman is in the intensive care unit immediately following removal
of a left adrenal pheochromocytoma. Her blood pressure is 80/40 mmHg. The
MOST appropriate treatment of the patient's hypotension is:
a. Epinephrine.
b. IV bolus of lactated Ringer solution.
c. Methylprednisolone.
d. Phenoxybenzamine.
e. Phenylephrine.
P a g e 17 | 25
ElEMAM’s SurgiNotes 2022
69.A 75-year-old woman presents with acute back pain after a minor fall.
Radiological examination reveals an osteoporotic crush fracture. What is the
expected laboratory finding?
a. Hypercalcaemia.
b. Hyperphosphatemia.
c. Hypokalaemia.
d. Hyponatraemia.
e. Normocalcaemia.
73.A complication that enteral and parenteral feeding have in COMMON is:
a. Increased incidence of sepsis.
b. Intestinal villous atrophy.
c. Elevated liver transaminases.
d. Hyperosmolar non-ketotic coma.
e. Diarrhea.
74.A 50-year-old male diabetic patient whose diabetes has been controlled by
regimen alone is going to have repair of an incisional hernia. What is your plan
of management?
a. Continuation of diet and determination of serum glucose level before
surgery.
b. Subcutaneous administration of regular insulin.
c. Oral hypoglycemic agents for 3 days atter surgery.
d. Insulin infusion beginning one for before surgery.
e. Increased oral carbohydrate intake to prevent ketosis.
P a g e 19 | 25
ElEMAM’s SurgiNotes 2022
79.A 50-year-old man with small-bowel fistula has been receiving TPN for the
previous 3 weeks. He is scheduled for exploratory laparotomy and closure of
fistula. On the day of surgery, TPN is discontinued and intravenous infusion with
Ringer's lactate is started. An hour later, the patient was found to be anxious,
sweating and tachycardic. What is the MOST likely cause?
a. Anxiety.
b. Hypoglycemia.
c. Hypovolemia.
d. Unexplained hemorrhage.
e. Hyperglycemia.
80.In which of the following conditions is the enteral route appropriate for
nutrition?
a. Upper gastrointestinal bleed.
b. Complete small bowel obstruction.
c. Acute flare up of Crohn's disease.
d. Low output colonic fistula.
e. High output small bowel fistula.
P a g e 20 | 25
ElEMAM’s SurgiNotes 2022
83.A 75-year-old female presents with ductal carcinoma in situ for simple
mastectomy. She has marked muscle wasting on examination and admits to a
poor diet. Which of the following values is MOST predictive of postoperative
mortality?
a. Serum sodium.
b. Serum albumin.
c. Serum protein.
d. Serum creatinine.
e. Serum glucose.
84.A patient who presented with severe diarrhea as a result of clostridium difficile,
develops a metabolic acidosis. This is MOST likely due to loss of which one of
the following ions in the diarrhea?
a. Cl-.
b. HCO3-.
c. K+.
d. NH4+.
e. OH-.
P a g e 21 | 25
ElEMAM’s SurgiNotes 2022
92.Which site or venue is the area that is at GREATEST risk for surgical errors?
a. Operating room.
b. Surgical intensive care unit.
c. Hospital wards/floors.
d. Emergency department.
e. Ambulatory care sites.
P a g e 23 | 25
ElEMAM’s SurgiNotes 2022
P a g e 24 | 25
ElEMAM’s SurgiNotes 2022
97. An 80-year-old woman with advanced Alzheimer's disease has been ill for 4 days
and is transferred from the nursing home with fever, hypotension and abdominal
swelling. Computed tomographic (CT) scan reveals a superior mesenteric artery
thrombosis, bowel ischemia and pneumatosis. She is acidotic and in acute renal
failure. Surgical intervention would necessitate resection of a significant length of
small and large bowel. Which of the following statements about this patient's care
is TRUE?
a. She is dying and palliative care is indicated.
b. The decision for further care should be left solely with the family.
c. Operation is the only chance for cure o the bowel ischemia and should be
performed.
d. Aggressive ICU resuscitation is warranted and operation should be
performed if the patient stabilizes.
e. Operation should be performed, but dialysis should not be contemplated.
98. Duty to intervene on the patient's behalf to increase comfort, health and well-
being:
a. Non-maleficence.
b. Autonomy.
c. Euthanasia.
d. Beneficence.
e. Justice.
99. Which of the following statements regarding the role of collagen in wound healing
is TRUE?
a. Collagen synthesis in the initial phase of injury is the sole responsibility of
endothelial cells.
b. Net collagen content increases for up to 2 years after injury.
c. At 3 weeks after injury, more than 50% of the tensile strength of the wound
has been restored.
d. Tensile strength of the wound increases gradually for up to 2-years after
injury; however, it generally reaches a level of only about 80% of that of
uninjured tissue
e. Tensile strength is the force necessary to reopen a wound.
100. Respiratory alkalosis can occur as a result of:
a. Asphyxia.
b. Asthma.
c. Severe emphysema.
d. Hyperventilation.
e. Hypoventilation.
P a g e 25 | 25
ElEMAM’s SurgiNotes 2022
11.A 30-year-old motorcyclist suffers a closed fracture to the mid-shaft of the tibia.
Anterior compartment syndrome could cause loss of sensation:
a. In the first web space.
b. Over the dorsum of the foot.
c. Over the lateral edge of the foot.
d. Over the medial aspect of the hallux.
e. Over the medial malleolus.
12.An 82-year-old man has complete occlusion of his inferior mesenteric artery on
angiography but no symptoms or signs of colonic ischemia. Which of the
following arteries is the MOST likely additional source of blood supply to the
territory of the inferior mesenteric artery?
a. Left colic.
b. Left gastroepiploic.
c. Middle colic.
d. Splenic.
e. Superior rectal.
P a g e 3 | 25
ElEMAM’s SurgiNotes 2022
14.A 22-year-old man is admitted to the emergency department after falling from
his bicycle. Radiograph examination reveals a fracture of the tibia above ankle.
MRI and physical examination reveal the tibial nerve is severed on the posterior
aspect of tibia. Which of the following signs will MOST likely present during
physical examination?
a. Sensory loss of the dorsum of the foot.
b. Sensory loss on the sole of the foot.
c. Foot drop.
d. Paralysis of the extensor digitorum brevis.
e. Sensory loss of the entire foot.
16.Which of the following is TRUE regarding the physiology of the thyroid gland?
a. Absorption of iodides occurs by passive diffusion.
b. Potassium perchlorate prevents the union between iodine and tyrosine.
c. T3 is four times more active than T4.
d. The half-life of thyroxine is 4 days.
e. The necessary intake of iodide is 50 ug/day.
P a g e 4 | 25
ElEMAM’s SurgiNotes 2022
18.Which of the following is the MOST potent stimulus for contraction of the gall
bladder?
a. Acetylcholine.
b. Cholecystokinin.
c. Intravenous hyperalimentation.
d. Secretin.
e. Vagal stimulation.
19.A 60-year-old male patient is suffering from diabetic ketoacidosis. Which of the
following is the MOST important buffer base in the extracellular fluid?
a. Bicarbonate.
b. Hemoglobin.
c. Lactate.
d. Phosphate.
e. Plasma proteins.
22.During the second (Proliferative) phase of wound healing the predominant cells
in the wound site are fibroblasts. Cross linkage of collagen requires hydroxy-
proline and hydroxy-lysine residues, which requires a specific vitamin to be
available in sufficient quantities. Deficiency of which vitamin results in collagen
that is unstable?
a. Vitamin B2 (Riboflavin).
b. Vitamin B6 (Pyridoxine).
c. Vitamin C (Ascorbic acid).
d. Vitamin D (Cholecalciferol).
e. Vitamin E (Tocopherol).
24.A 70-year-old man with chronic obstructive pulmonary disease is admitted for
elective hemicolectomy. What is a preoperative arterial blood gas analysis likely
to show?
Bicarbonate Arterial pCO2
Decreased Decreased A
Increased Decreased B
Normal Decreased С
Decreased Increased D
Increased Increased E
P a g e 6 | 25
ElEMAM’s SurgiNotes 2022
29.A 50-year-old male patient was punctured in his foot by a very tiny piece of glass
which could not be removed. Three months later the patient presented by a
firm small nodule at the site of the puncture and this was excised. Which of the
following cells will be predominant on pathological examination of the removed
nodule?
a. Epithelioid cells.
b. Fibroblasts.
c. Lymphocytes.
d. Plasma cells.
e. Polymorphonuclear leucocytes.
30.In patients with reflux esophagitis, What does the presence of columnar cells in
the esophageal mucosa represent?
a. Carcinoma in situ.
b. Carcinoma.
c. Dysplasia.
d. Hyperplasia.
e. Metaplasia.
31.With regard to protein loss after injury, which of the following statements is
TRUE?
a. It can be prevented by total parenteral nutrition.
b. It occurs primarily from skeletal muscle.
c. It occurs primarily from the site of injury.
d. It results from significant decrease intake.
e. It results from impaired synthesis.
P a g e 8 | 25
ElEMAM’s SurgiNotes 2022
P a g e 10 | 25
ElEMAM’s SurgiNotes 2022
P a g e 11 | 25
ElEMAM’s SurgiNotes 2022
45.Which of the following which is the MOST effective way in preventing surgical
infection?
a. Antibiotic prophylaxis.
b. Bowel preparation.
c. Drains and irrigation.
d. Skin preparation.
e. Surgical technique.
47.A 37-year-old woman with end-stage renal disease had a cadaveric renal
transplant 18 months previously. She tolerated her transplantation well and has
been receiving a stable regiment of tacrolimus and corticosteroids with a
creatinine of 1.0 mg/dl. She now presents with fatigue, bone pain and
osteoporosis. Her serum calcium is 11.5 mg/dl. The next step in management
should be:
a. Subtotal parathyroidectomy.
b. An increase in corticosteroid dose.
c. Hydration and diuresis.
d. Calcitonin injections.
e. Phosphate binders.
51.The arterial blood gas analysis of a patient was pH 7.5, P(CO2) 47 mmHg, HCO3-
35 mmol/L. This patient MOST likely is suffering from:
a. Chronic obstructive pulmonary disease.
b. Diabetic ketoacidosis.
c. Persistent diarrhea.
d. Profound vomiting.
e. Salicylate poisoning.
53.In the presence of acute blood loss, what is the initial mechanism to maintain
an adequate preload to the heart?
a. Development of tachycardia.
b. Hormonal effects of angiotensin.
c. Hormonal effects of rennin.
d. Increase in systemic vascular resistance.
e. Increased cortisol secretion.
55.Following an operation and extubation a patient was restless. His arterial blood
gases revealed: pH 7.36; PaO2 65 mmHg; PaCO2 55 mmHg; PaCO3- 30 mmol/L.
The physiologic status can BEST be described as which of the following?
a. Respiratory alkalosis.
b. Respiratory acidosis.
c. Metabolic acidosis.
d. Metabolic alkalosis.
e. Combined respiratory and metabolic acidosis.
58.A 21-year-old man undergoes major abdominal surgery after a motor vehicle
collision. He has a cardiac arrest in the intensive care unit shortly after returning
from surgery. Select the MOST appropriate pharmacologic agent for the patient:
a. Epinephrine.
b. Norepinephrine.
c. Phenylephrine.
d. Dopamine.
e. Dobutamine.
62.A 55-year-old male patient is receiving 150 mg of aspirin daily because he has
coronary artery disease. Which of the following tests will be affected in this
patient?
a. Bleeding time.
b. Coagulation time.
c. INR.
d. PTT.
e. Thrombin time.
64.Which of the following is the MOST likely cause of increased prothrombin time?
a. Christmas disease.
b. Hemophilia.
c. Heparin therapy.
d. Von Willebrand’s disease.
e. Warfarin overdose.
P a g e 16 | 25
ElEMAM’s SurgiNotes 2022
69.What does increased level of fibrin degradation products (FDP) in the blood
denote?
a. Diminished fibrinogen synthesis.
b. Diminished platelet count.
c. Diminished platelet function.
d. Disseminated intravascular coagulation (DIC).
e. Heparin overdose.
71.Which of the following is the BEST method of identifying the liability to bleeding
during a surgical procedure?
a. Platelet count.
b. A complete history and physical examination.
c. Bleeding time.
d. Lee-White clotting time.
e. Prothrombin time (PT).
75.A 45-year-old woman with deep vein thrombosis is taking warfarin (Coumadin),
5 mg/d. Seven days after initiation of therapy, she has warfarin-induced skin
necrosis. Which of the following statements regarding this condition is TRUE?
a. It commonly occurs after warfarin therapy.
b. It usually involves the upper extremities.
c. It improves with an increase in the dose of Coumadin.
d. It improves with a decrease in the dose of Coumadin.
e. It requires cessation of Coumadin and infusion of heparin.
76.Which of the following is MOST likely to be a risk factor for the development of
carpal tunnel syndrome?
a. Addison's syndrome.
b. Diabetes insipidus.
c. Fibromyalgia.
d. Graves' disease.
e. Hypothyroidism.
P a g e 19 | 25
ElEMAM’s SurgiNotes 2022
77.A 30-year-old female patient complains of fatigue, generalized bony aches and
depression. The patient gives a history of 2 previous operations for urinary
stones. Which of the following laboratory tests is MOST accurate in the
diagnosis?
a. 24 hours urinary calcium.
b. Ionizable serum calcium.
c. Parathormone hormone.
d. Plasma chloride.
e. Serum phosphate.
79.Which of the following is the MOST potent stimulus for aldosterone secretion?
a. ACTH.
b. Antidiuretic hormone.
c. Hyperkalaemia.
d. Hypernatraemia.
e. The rennin angiotensin system.
82.A 40-year-old woman had the anterior lobe of the pituitary removed because of
a tumor. Without postoperative supplements, which of the following could
occur?
a. Failure to produce adequate amounts of thyroxine.
b. Failure to produce parathyroid hormone in response to hypocalcaemia.
c. Failure to secrete catecholamines in response to stress.
d. Failure to secrete insulin in hyperglycaemia.
e. Inability to concentrate urine in response to water deprivation.
85. Which of the following biochemical tests for the work-up of pheochromocytomas
has the HIGHEST sensitivity?
a. Urinary norepinephrine.
b. Urinary vanillylmandelic acid.
c. Urinary total metanephrine.
d. Plasma free metanephrines and normetanephrine.
e. Urinary epinephrine.
86. Regarding the anatomy of the thyroid gland, which of the following is TRUE?
a. In about 80% of persons, the recurrent laryngeal nerve traverses anterior to
the inferior thyroid artery.
b. The recurrent laryngeal nerve has an oblique course around the subclavian
artery on the left side.
c. The superior laryngeal nerve provides both sensory and motor function to
the larynx.
d. The thyroid gland is innervated only by parasympathetic fibers from the
vagus nerve.
e. Unilateral recurrent laryngeal nerve injury usually results in airway
compromise that necessitates tracheotomy.
88. A 47-year-old male with Crohn's colitis maintained on 40 mg prednisone daily for
the past year presents for elective colectomy. The procedure was uncomplicated
and he was adequately resuscitated. In the post-anesthesia care unit (PACU), the
patient is noted to be febrile and hypotensive with mean arterial blood pleasure of
80/60 mmHg in the 50s. What is your next step in management?
a. IV dobutamine.
b. Hydrocortisone.
c. 1 unit of packed red blood cells.
d. Antibiotics.
e. Epinephrine.
P a g e 22 | 25
ElEMAM’s SurgiNotes 2022
92.A 50-year-old man with small-bowel fistula has been receiving TPN for the
previous 3 weeks. He is scheduled for exploratory laparotomy and closure of
fistula. On the morning of the day of surgery, TPN is discontinued and
intravenous infusion with Ringer's lactate is started. An hour later, the patient
is found to be anxious, sweating and tachycardic. What is the MOST likely cause?
a. Anxiety.
b. Hypoglycemia.
c. Hypovolemia.
d. Unexplained hemorrhage.
e. Hyperglycemia.
P a g e 23 | 25
ElEMAM’s SurgiNotes 2022
93.A patient of serious car accident remains intubated and is without enteral
feeding for 3 days. Which of the following is the MAIN energy source during
critical illness/injury?
a. Skeletal muscle.
b. Liver.
c. Adipose tissue.
d. Kidney.
e. Gut.
95.A patient with multiple small bowel resections for Crohn's disease presents with
an acute flare. He is suspected of having functionally a short gut syndrome and
started on TPN. Refeeding syndrome is characterized by which of the following
electrolyte abnormalities?
a. Hyponatremia, hypokalemia and hypercalcemia.
b. Hyperphosphatemia, hypokalemia and hypocalcaemia.
c. Hypokalemia, hypomagnesemia and hypophosphatemia.
d. Hypokalemia, hyponatremia and hypomagnesemia.
e. Hyperkalemia, hypernatremia and hypercalcemia.
96.After 4 days of TPN, a patient above develops blood glucose levels greater than
300 mg/dl. Hyperglycemia in a surgical patient receiving TPN may BEST be
managed by:
a. Oral hypoglycemics.
b. Decreasing the dextrose load and doubling the amount of fat.
c. Adding regular insulin to the TPN.
d. Discontinuing TPN for 2 weeks and then trying to start TPN again.
e. Increasing the concentration of protein and carbohydrate calories and
decreasing that of lipids.
P a g e 24 | 25
ElEMAM’s SurgiNotes 2022
P a g e 25 | 25
ElEMAM’s SurgiNotes 2022
P a g e 1 | 25
ElEMAM’s SurgiNotes 2022
P a g e 2 | 25
ElEMAM’s SurgiNotes 2022
P a g e 3 | 25
ElEMAM’s SurgiNotes 2022
14.The femoral canal to a point of weakness in the abdominal wall. Which of the
following statements is the MOST accurate regarding the anatomy of the
femoral canal?
a. The femoral artery forms the anterior border of the femoral canal.
b. The formal vein forms the lateral border.
c. The inguinal ligament lies posterior to the canal.
d. The pectineal pant of the inguinal ligament is anterior to the canal.
e. The pectineal ligament forms the medial border.
P a g e 4 | 25
ElEMAM’s SurgiNotes 2022
19.Which of the following is the MOST potent stimulus for ADH hormone
secretion?
a. Decreased plasma osmolality.
b. Decreased plasma volume.
c. Hypothalamic releasing factor.
d. Increased plasma osmolality.
e. Increased plasma volume.
20.Which of the following is the MOST potent stimulus for contraction of the gall
bladder?
a. Acetylcholine.
b. Cholecystokinin.
c. Intravenous hyperalimentation.
d. Secretin.
e. Vagal stimulus.
P a g e 5 | 25
ElEMAM’s SurgiNotes 2022
21.A 60-year old male patient is suffering from diabetic ketoacidosis. Which of the
following is the MOST powerful buffer base in the extracellular fluid?
a. Bicarbonate.
b. Haemoglobin.
c. Lactate.
d. Phosphate.
e. Plasma proteins.
22.The following acid-base data: pH: 7.2, PCO2: 20 mmHg, HCO3-: 8 mmol/L, Base
excess: -19 mmol/L, would be MOST consistent with:
a. Anxiety.
b. Lobar collapse of the lung.
c. Pyloric obstruction.
d. Septic shock.
e. Starvation.
23.Which of the following is the MOST potent stimulus for aldosterone synthesis?
a. ACTH.
b. Antidiuretic hormone (ADH).
c. Hyperkalemia.
d. Hypernatraemia.
e. Renin angiotensin system.
24.The patient whose blood pH is 7.47, whose PCO2 is 31 mmHg in the arterial
blood and whose levels of bicarbonate ion in arterial blood is 23 mEq/L is in:
a. Compensated metabolic alkalosis.
b. Uncompensated respiratory acidosis.
c. Uncompensated respiratory alkalosis.
d. Uncompensated metabolic acidosis.
e. Uncompensated metabolic alkalosis.
P a g e 6 | 25
ElEMAM’s SurgiNotes 2022
25.A 25-year old man presents to the emergency department after being involved
in a road traffic accident. He is conscious, talking, has a blood pressure of 90/70
mmHg and heart rate of 100 beats/minute. He has sustained a fracture pelvis
and femur. Which of the following is TRUE?
a. His total peripheral resistance is decreased.
b. The discharge rate of his carotid sinus nerves is increased.
c. There is a decrease in renin production.
d. There is an increase in angiotensin II.
e. There is an increase in renal sodium excretion.
26.Which of the following is the MOST important stimulus for triggering the
endocrine response after injury?
a. Afferent nerve stimuli from the injured area.
b. Hypovolemia.
c. Tissue acidosis.
d. Local wound factors.
e. Temperature changes.
31.A patient who presented with severe diarrhea as a result of Clostridium difficile,
develops a metabolic acidosis. This is MOST likely due to loss of which one of
the following ions in the diarrhoea?
a. Cl-.
b. HCO3-.
c. K+.
d. NH4+.
e. OH-.
33.In patients with reflux esophagitis. What does the presence of columnar cells in
the esophageal mucosa represent?
a. Carcinoma in situ.
b. Carcinoma.
c. Dysplasia.
d. Hyperplasia.
e. Metaplasia.
34.A patient who has hydatid cyst of the liver will have increased percentage of
which cells in his blood count:
a. Basophils.
b. Eosinophils.
c. Lymphocytes.
d. Monocytes.
e. Neutrophils.
35.With regard to protein loss after injury, which of the following statements is
TRUE?
a. It can be prevented by total parenteral nutrition.
b. It occurs primarily from skeletal muscle.
c. It occurs primarily from the site of injury.
d. It results from impaired synthesis.
e. It results from significant decrease intake.
39.A 45-year-old diabetic male patient complains of severe pain in the thigh.
Examination reveals spreading oedema, swelling and tenderness of the thigh. A
provisional diagnosis of necrotizing fasciitis is made. Which of the following is
MOST important in the treatment?
a. Antifungal agents.
b. Antitoxin.
c. Hyperbaric O2.
d. Immunoglobulins.
e. Wide surgical debridement.
P a g e 10 | 25
ElEMAM’s SurgiNotes 2022
P a g e 11 | 25
ElEMAM’s SurgiNotes 2022
45.An 80-year-old woman who lives in a nursing home and who had just finished a
10-days course of antibiotics, has abdominal pain and profuse diarrhea. Her
stool is tested and comes back positive for Cl. difficile. What is the MOST
appropriate initial management?
a. Oral vancomycin.
b. Intravenous vancomycin.
c. Metronidazole.
d. Vancomycin enema.
e. Supportive treatment only.
46.Which of the following statement about the usage of antibiotics for the
prevention of surgical site infection is NOT TRUE?
a. Antibiotics should be administrated within 60 minutes of incision.
b. The therapeutic dose of the antibiotic should be administrated
intravenously.
c. Adequate tissue concentrations should be maintained during operation
by re-dosing as necessary.
d. Antibiotics should be continued for 48 hours after operation.
e. Vancomycin is the agent of chronic for patients from nursing homes
undergoing hip replacement.
55.The STRONGEST independent risk factor for stress-related mucosal damage and
gastrointestinal bleeding in critically ill patients is:
a. Glucocorticoid administration.
b. Hypotension.
c. Mechanical ventilator support for 48 or more hours.
d. Organ transplantation.
e. Renal failure.
60. A patient who had gastrectomy 10 days ago developed massive pulmonary
embolism proved by CT angiography. What is the recommended treatment?
a. IV heparin.
b. Fibrinolytic agent.
c. Warfarin.
d. IV acetylsalicylic acid.
e. Corticosteroids.
P a g e 15 | 25
ElEMAM’s SurgiNotes 2022
62.A 70-year-old man with chronic obstructive pulmonary disease is admitted for
elective hemicolectomy. What is a preoperative arterial blood gas analysis likely
to show?
Arterial PCO2 Bicarbonate.
a. Decreased Decreased.
b. Decreased Increased.
c. Decreased Normal.
d. Increased Decreased.
e. Increased Increased.
63.A 39-year-old woman is making a slow but adequate recovery after sustaining a
40% surface area burn injury. On the sixth postoperative day she becomes
unwell. She vomits intermittently, has painless abdominal distention and starts
to hiccup. What is the MOST likely cause of these symptoms?
a. Acute gastric dilatation.
b. Acute intestinal obstruction.
c. Clostridium difficile infection.
d. Faecal impaction.
e. Systemic sepsis.
64.A 78-year-old man with a history of coronary artery disease requests an elective
epigastric hernia repair. Which of the following is valid reason for delaying the
proposed surgery?
a. Coronary artery bypass surgery 3 months earlier.
b. A history of cigarette smoking.
c. Jugular venous distension.
d. Hypertension.
e. Hyperlipidemia.
P a g e 16 | 25
ElEMAM’s SurgiNotes 2022
66.Which of the following types of shock is associated with high pulmonary wedge
pressure?
a. Hypovolemic shock.
b. Cardiogenic shock.
c. Early septic shock.
d. Late septic shock.
e. Neurogenic shock.
69.The anion gap will increase with an increase in the plasma concentration of:
a. Sodium.
b. Potassium.
c. Chloride.
d. Bicarbonate.
e. Lactate.
70.A 55-year-old male patient is receiving 150 mg of aspirin daily because he has
coronary artery disease. Which of the following tests will be affected in this
patient?
a. Bleeding time.
b. Coagulation time.
c. INR.
d. PIT.
e. Thrombin time.
71.A 30-year-old male patient was admitted to the casualty department due to a
car accident. The patient had fracture of the pelvis and the right femur and he
received 5 liters of blood following which he started to have bleeding from nose
and mouth. What is the MAIN cause of this bleeding tendency?
a. Decrease in fibrinogen.
b. Decrease in prothrombin.
c. Decrease of calcium.
d. Increased fibrinolytic activity.
e. Platelet depletion.
72.A patient with a life threatening pulmonary embolus is receiving heparin. She
developed serious vaginal bleeding and a major retroperitoneal haematoma
after 5 days of heparin therapy. What is the recommended treatment?
a. Reverse heparin and evacuate the haematoma.
b. Reverse heparin by protamine sulphate and insert a vena caval filter.
c. Stop heparin and closely observe the patient.
d. Stop heparin, give fresh frozen plasma and start warfarin therapy.
e. Switch to low-dose heparin.
P a g e 18 | 25
ElEMAM’s SurgiNotes 2022
74.You were obliged to operate for a strangulated inguinal hernia in a patient who
was receiving 300 mg aspirin daily. During surgery there was excessive bleeding.
What would you advise?
a. Desmopressin.
b. Fresh blood.
c. Fresh frozen plasma.
d. Fresh platelets.
e. IV vitamin K.
75.What in the cause of the prolonged prothrombin time in patients with liver
cirrhosis?
a. Abnormal fibrinolysis.
b. Deficiency of protein S.
c. Factor VII deficiency.
d. Thrombocytopenia.
e. Von Willebrand's disease.
82.A 50-year-old female patent has chronic renal failure and has been maintained
on chronic dialysis. The patient underwent cholecystectomy. Post-operatively
she had severe bleeding. What is the MOST likely cause for this bleeding?
a. Elevated PT.
b. Elevated PTT.
c. Low platelet count.
d. Decreased platelet aggregation.
e. Sepsis.
83.A mother mentions that her 8-year-old boy gets recurrent attacks of
hemarthrosis following a minimal trauma. Investigations revealed normal
platelet count and prothrombin time, but the partial thromboplastin time is
prolonged. Which of the following statements regarding this clinical condition is
TRUE?
a. There is an underlying liver problem.
b. The sisters of this boy are usually having the same problem.
c. The boy has had repeated episodes of epistaxis.
d. There is no family history in this condition.
e. Transfusion of factor VIII concentrate is helpful.
84.70-year-old female patient is receiving warfarin because she has AF and had
previous thrombosis. Which of the following statements regarding warfarin is
CORRECT?
a. The dose of warfarin is adjusted according to the partial thromboplastin
time.
b. Warfarin takes about 8 hours to exert its effects.
c. Warfarin acts by inhibiting factor XII.
d. If the patient is also taking aspirin, the dose of warfarin should be
reduced.
e. Protamine sulphate is the antidote to warfarin.
P a g e 21 | 25
ElEMAM’s SurgiNotes 2022
85.Which of the following is MOST likely to be a risk factor for the development of
carpal tunnel syndrome?
a. Addison's syndrome.
b. Diabetes insipidus.
c. Fibromyalgia.
d. Graves' disease.
e. Hypothyroidism.
86.Regarding the anatomy of the thyroid gland, which of the following is TRUE?
a. In about 80% of persons, the recurrent laryngeal nerve traverses anterior
to the inferior thyroid artery.
b. The recurrent laryngeal nerve has an oblique course around the
subclavian artery on the left side.
c. The superior laryngeal nerve provides both sensory and motor function
to the larynx.
d. The thyroid gland is innervated only by parasympathetic fibers from the
vagus nerve.
e. Unilateral recurrent laryngeal nerve injury usually results in airway
compromise that necessitates tracheotomy.
90.A 55-year-old female …… Grave’s disease and has been treated by propyl
thiouracil for 4 years. The patient developed mass enlargement of the thyroid
gland. What is the cause of this enlargement?
a. ……
b. ……
c. ……
d. ……
e. ……
92.A 55-year-old female patient who had metastatic breast cancer presents with
weakness, anorexia, malaise, constipation and back pain and lethargy.
Laboratory studies include a normal chest X-Ray; serum albumin 3.2 mg/dl;
serum calcium 14 mg/dl; serum phosphorus 2.6 mg/dl; serum chloride 108
mg/dl; BUN 32 mg/dl and creatinine 2.0 mg/dl. What is the appropriate initial
management?
a. Intravenous normal saline infusion.
b. Administration of thiazide diuretics.
c. Administration of intravenous phosphorus.
d. Use of mithramycin.
e. Neck exploration and parathyroidectomy.
P a g e 23 | 25
ElEMAM’s SurgiNotes 2022
P a g e 24 | 25
ElEMAM’s SurgiNotes 2022
100. World health organization established the surgical safety checklist in order
to maintain patient safety in the operating theatre. One of the measures that
eliminate wrong site surgery is to perform a time-out check. The timing of this
check is:
a. Before referral to the operating theatre.
b. Before induction of anesthesia.
c. Before skin incision.
d. Before removal of important organ.
e. Before closure of the wound.
P a g e 25 | 25
ElEMAM’s SurgiNotes 2022
2. Which of the following statement is TRUE regarding surgical excision of the right
submandibular salivary gland?
a. Injury to the lingual nerve would result in loss of sensations to the posterior
one third of the right side of the tongue.
b. Injury to the hypoglossal nerve would result in deviation of the tongue to the
right side on protrusion of the tongue.
c. The great auricular nerve is at a risk of injury.
d. Injury of the marginal mandibular branch of the facial nerve would result in
sensory loss at the angle of the mandible.
e. Injury to the cervical branch of the facial nerve would result in drooping of the
right angle of the mouth.
3. Which of the following is the sensory supply to the skin of the tip of the index finger?
a. Radial nerve only.
b. Radial and median nerves.
c. Median and ulnar nerves.
d. Ulnar nerve only.
e. Median nerve only.
4. Which of the following statements is TRUE regarding the marginal mandibular nerve?
a. Is a branch of the 5th cranial nerve.
b. Supplies motor function to the strap muscles.
c. May be associated with the submandibular gland.
d. Lies deep to the facial vein.
e. Is most often injured during dissection of the posterior triangle (Level 5) of the
neck.
P a g e 1 | 25
ElEMAM’s SurgiNotes 2022
10.A 21-year-old man has been stabbed in the back of the knee, dividing the
popliteal artery and is undergoing repair via a posterior approach. Which of the
following structures is MOST likely to be encountered first when dissecting deep
from the skin incision?
a. Popliteal artery.
b. Popliteus muscle.
c. Popliteal vein.
d. Tibial nerve.
e. Soleus muscle.
14.While assessing an elderly woman with severe neck arthritis you note she has
weakness of finger abduction and adduction. This is MOST likely to be due to
compression of which spinal nerve root of the brachial plexus?
a. Fifth cervical.
b. Sixth cervical.
c. Seventh cervical.
d. Eighth cervical.
e. First thoracic.
20.A 60-year-old male patient is suffering from diabetic ketoacidosis. Which of the
following is the MOST important buffer base in the extracellular fluid?
a. Bicarbonate.
b. Haemoglobin.
c. Lactate.
d. Phosphate.
e. Plasma proteins.
P a g e 5 | 25
ElEMAM’s SurgiNotes 2022
21.The following acid-base data: pH: 7.2, PCO2: 20 mmHg, HCO3-: 8 mmol/L, Base
excess: -19 mmol/L, would be MOST consistent with:
a. Anxiety.
b. Lobar collapse of the lung.
c. Pyloric obstruction.
d. Septic shock.
e. Starvation.
25. Which of the following is TRUE regarding the release of tumor necrosis factor-α?
a. Can be effectively blocked by anti-TNF-a antibodies to halt systemic
inflammatory response syndrome (SIRS).
b. Does not have any beneficial effects in the early phases of the inflammatory
response.
c. Is primarily from leukocytes.
d. Promotes polymorphonuclear (PMN) cell adherence and further cytokines
release.
e. Is always deleterious.
26. A 21-year-old graduate student has a large hypertrophic scar on the lower part of
her face. The patient had sustained a laceration on her face 2 years previously after
injuring her face. Which of the following statements regarding scar revision is
TRUE?
a. Scar maturation refers to the change in size of the wound in the first 1 to 2
months.
b. Scar revision should have been performed in the first 3 months after injury
to minimize fibrosis.
c. Revision should be performed earlier in children than in adults.
d. It corrects undesirable pigmentation.
e. Scar revision should be delayed approximately 1 year to allow maturation.
27. A 25-year-old man presents to the emergency department after being involved in
a road traffic accident. He is conscious, talking, has a blood pressure of 90/70
mmHg and a heart rate of 100 beats/minute. He has sustained a fractured pelvis
and femur. Which of the following is TRUE?
a. His total peripheral resistance is decreased.
b. The discharge rate of his carotid sinus nerves is increased.
c. There is a decrease in renin production.
d. There is an increase in angiotensin II.
e. There is an increase in renal sodium excretion.
28. Which one of the following findings is MOST likely to be present in a patient with
severe magnesium deficiency?
a. Respiratory depression.
b. Bradycardia.
c. Tetany.
d. Hypotension.
e. Loss of patellar reflex Hyperactive reflexes.
P a g e 7 | 25
ElEMAM’s SurgiNotes 2022
30.A 50-year-old male patient was punctured in his foot by a very tiny piece of glass
which could not be removed. Three months later the patient presented by a
firm small nodule at the site of the puncture and this was excised. Which of the
following cells will be predominant on pathological examination of the removed
nodule?
a. Epithelioid cells.
b. Fibroblasts.
c. Lymphocytes.
d. Plasma cells.
e. Polymorphonuclear leucocytes.
33.In patients with reflux esophagitis. What does the presence of columnar cells in
the esophageal mucosa represent?
a. Carcinoma in situ.
b. Carcinoma.
c. Dysplasia.
d. Hyperplasia.
e. Metaplasia.
36.In the presence of acute inflammation, when an abscess has formed, which of
the following cell types is MOST likely to be primarily involved in phagocytic
activity?
a. Erythrocytes.
b. Lymphocytes.
c. Macrophages.
d. Mast cells.
e. Plasma cells.
P a g e 9 | 25
ElEMAM’s SurgiNotes 2022
37.Which of the following is the MOST likely cause of pyrexia occurring 48 hours
after an abdominal operation?
a. Chest infection.
b. DVT.
c. Leaking intestinal anastomosis.
d. Pulmonary embolism.
e. Wound infection.
39.A 45-year-old diabetic male patient complains of severe pain in the thigh.
Examination reveals spreading oedema, swelling and tenderness of the thigh. A
provisional diagnosis of necrotizing fasciitis is made. Which of the following is
MOST important in the treatment?
a. Antifungal agents.
b. Antitoxin.
c. Hyperbaric O2.
d. Immunoglobulins.
e. Wide surgical debridement.
47.An obese 50-year-old man who has had a renal transplant undergoes left direct
hernia repair with mesh under local anesthesia. Medications include tacrolimus
(Prograf) and prednisone. A nasal swab is positive for methicillin-resistant
staphylococcus aureus (MRSA). Which of the following statements is TRUE?
a. His wound classification is clean.
b. He should receive oral antibiotics for 48 hours postoperatively.
c. His risk of surgical site infection is less than 1%.
d. Mesh is contraindicated in this patient.
e. The wound should be irrigated with vancomycin.
48.Compared with wet to dry dressing changes, negative pressure wound therapy
is associated with all of the following EXCEPT:
a. Increased local blood flow.
b. Stimulation of granulation tissue.
? Wrong Q.
c. Decreased tissue edema.
d. Decreased bacterial counts.
e. Increased rate of wound healing.
P a g e 12 | 25
ElEMAM’s SurgiNotes 2022
60.A 24-year-old woman has acute renal failure following postpartum hemorrhage.
Laboratory studies showed serum glucose, 150 mg/dL: sodium, 135 mEq/L:
potassium, 6.5 mEq/L: chloride, 105 mEq/L and bicarbonate, 15 mEq/L. Which
of the following is recommended?
a. Decrease potassium chloride to 10 mEq/L.
b. Intravenous 0.9% sodium chloride.
c. 100 ml of 50% glucose water with 10 U insulin.
d. Intravenous calcitonin.
e. Intravenous magnesium sulfate.
P a g e 15 | 25
ElEMAM’s SurgiNotes 2022
61.Following an operation and extubation a patient was restless. His arterial blood
gases revealed: pH 7.36: PaO2 65 mmHg: PaCO2 55 mmHg: PaCO3- 30 mmol/L.
The physiologic status can BEST be described as which of the following?
a. Respiratory alkalosis.
b. Respiratory acidosis.
c. Metabolic acidosis.
d. Metabolic alkalosis.
e. Combined respiratory and metabolic acidosis.
64.A 40-year-old man is confused and restless the second day after upper
abdominal surgery and repair of a hiatus hernia. What is the MOST probable
cause of his condition?
a. Pulmonary embolism.
b. Narcotic overdose.
c. Pulmonary atelectasis.
d. Electrolyte imbalance.
e. Starvation ketosis.
P a g e 16 | 25
ElEMAM’s SurgiNotes 2022
65.A 70-year-old man with chronic obstructive pulmonary disease is admitted for
elective hemicolectomy. What is a preoperative arterial blood gas analysis likely
to show?
Arterial PCO2 Bicarbonate.
a. Decreased Decreased.
b. Decreased Increased.
c. Decreased Normal.
d. Increased Decreased.
e. Increased Increased.
66.A 60-year-old man had undergone exploratory laparotomy for perforated
gastric ulcer with severe peritoneal contamination. Six hours after surgery, he is
tachycardic, hypertensive and has shallow respirations. Intubation and
institution of ventilatory support is indicated in the presence of which of the
following?
a. Respiratory rate of 23 breaths/min.
b. PaCO2 of 45 mmHg.
c. PaO2 of 55 mmHg on room air.
d. Heart rate of 140 bpm.
e. BP of 100/70 mmHg.
67.A seventy year-old man presents to the ER with several bone fractures and a
ruptured spleen after failing from a height. Which of the following BEST
represents his response to injury?
a. Decreased liver gluconeogenesis.
b. Inhibition of skeletal muscle breakdown by interleukin-1 and tumor
necrosis factor (TNF, cachectin).
c. Decreased urinary nitrogen loss.
d. Decreased glutamine consumption by fibroblasts, lymphocytes and
intestinal epithelial cells.
e. Hepatic synthesis of acute-phase reactants.
68.The arterial blood gases of a patient who had an operation for ruptured aortic
aneurysm are: pH 7.54, PO2 100 mmHg, PCO2 30 mmHg, HCO3- 30 mmol/L. What
is the acid base disturbance?
a. Respiratory acidosis.
b. Respiratory alkalosis.
c. Metabolic alkalosis.
d. Combined respiratory and metabolic alkalosis.
e. Compensated respiratory acidosis.
P a g e 17 | 25
ElEMAM’s SurgiNotes 2022
69.Which of the following types of shock is associated with high pulmonary wedge
pressure?
a. Hypovolemic shock.
b. Cardiogenic shock.
c. Early septic shock.
d. Late septic shock.
e. Neurogenic shock.
73. A 38-year-old man in end-stage renal failure resulting from polycystic kidney
disease receives a cadaveric renal transplant. Good renal function is established
but four weeks later deteriorates, the serum creatinine rising by 25%. Which of the
following processes is MOST likely to be responsible for this deterioration?
a. B-cell mediated rejection.
b. Circulating immune complex disease.
c. IgG antibody mediated rejection.
d. Post-transplant lymphoproliferative disorder.
e. T-cell mediated rejection.
74. A 78-year-old woman with emphysema receiving 28% oxygen by mask has the
following blood gas results:
pH PO2 PaCO2 Bicarbonate Base excess
Finding: 7.28 70 mm Hg 48 mm Hg 36 mmol/L +5
Normal: 7.35-7.45 90-110 35-45 22-26 -2 to +2
The MOST likely interpretation is:
a. Mixed respiratory and metabolic acidosis.
b. Partially compensated metabolic acidosis.
c. Partially compensated respiratory acidosis.
d. Uncompensated metabolic acidosis.
e. Uncompensated respiratory acidosis.
75. An elderly diabetic patient who has acute cholecystitis is found to have a serum
sodium level of 122 mEq/L and blood glucose of 600 mg/dl. After correction of the
glucose concentration to 120 mg/dl with insulin, the serum sodium concentration
would:
a. Decrease significantly unless patient receive normal saline.
b. Decrease transiently but return to 122 mEq/L.
c. Remain essentially unchanged.
d. Increase to normal range without specific therapy.
e. Increase to hypernatremic level.
76. A 60-year-old man with a past history of angina undergoes an uncomplicated
operation for an inguinal hernia. Postoperatively he is found to be hypotensive,
tachycardic and has a raised jugular venous pressure. What is the MOST likely
underlying cause of his hypotension?
a. Reduced afterload.
b. Reduced parasympathetic tone.
c. Reduced preload.
d. Reduced stroke volume.
e. Reduced sympathetic tone.
P a g e 19 | 25
ElEMAM’s SurgiNotes 2022
78.A 55-year-old male patient is receiving 150 mg of aspirin daily because has
coronary artery disease. Which of the following tests will be affected this
patient?
a. Bleeding time.
b. Coagulation time.
c. INR.
d. PTT.
e. Thrombin time.
88.A 50-year-old female patient has chronic renal failure and has been maintained
an chronic dialysis. The patient underwent cholecystectomy. Post-operatively
she had severe bleeding. What is the MOST likely cause for this bleeding?
a. Elevated PT.
b. Elevated PTT.
c. Low platelet count.
d. Decreased platelet aggregation.
e. Sepsis.
P a g e 22 | 25
ElEMAM’s SurgiNotes 2022
89. A 22-year-old man is brought into the emergency department in profound shock
after a fall from the fourth floor of a building. After resuscitation, small bowel
resection and hepatic segmentectomy are performed at laparotomy. He receives
15 units of packed RBCs, 4 units of fresh-frozen plasma and 8 L of Ringer's lactate.
On closure, diffuse oozing of blood is noted. What is the MOST likely cause?
a. Hepatic failure.
b. Hypersplenism.
c. Platelet deficiency.
d. Factor IX (Christmas factor) deficiency.
e. Congenital hypoprothrombinemia.
90. A mother mentions that her 8-year-old boy gets recurrent attacks of hemarthrosis
following a minimal trauma. Investigations revealed normal platelet count and
prothrombin time, but the partial thromboplastin time is prolonged. Which of the
following statements regarding this clinical condition is TRUE?
a. There is an underlying liver problem.
b. The sisters of this boy are usually having the same problem.
c. The boy has had repeated episodes of epistaxis.
d. There is no family history in this condition.
e. Transfusion of factor VIII concentrate is helpful.
91. A 70-year-old female patient is receiving warfarin because she has AF and had
previous thrombosis. Which of the following statements regarding warfarin is
CORRECT?
a. The dose of warfarin is adjusted according to the partial thromboplastin
time.
b. Warfarin takes about 8 hours to exert its effects.
c. Warfarin acts by inhibiting factor XII.
d. If the patient is also taking aspirin, the dose of warfarin should be reduced.
e. Protamine sulphate is the antidote to warfarin.
92. A 12-year-old boy with-known factor VIII deficiency has a painful, swollen,
immobile right knee. The clinician suspects hemarthrosis. Therapeutic options
include which of the following?
a. Immediate aspiration and compression dressings to prevent cartilage
necrosis.
b. Compression dressings and immobilization to prevent further bleeding.
c. Immediate aspiration after appropriate factor VIII replacement therapy.
d. Initial trial of factor VIII therapy, compression dressings, cold packs and rest
followed by active range-of-motion exercises.
e. Factor VIII replacement followed by arthrotomy.
P a g e 23 | 25
ElEMAM’s SurgiNotes 2022
93.The MOST useful laboratory test to assess both risk of bleeding and response to
therapy in patients with uremia is:
a. Bleeding time.
b. Platelet count.
c. INR.
d. APTT.
e. Thrombin time.
95.A 30-year-old female patient complains of fatigue, generalized bony aches and
depression. The patient gives a history of 2 previous operations for urinary
stones. Which of the following laboratory tests is MOST accurate in the
diagnosis?
a. 24 hours urinary calcium.
b. Ionizable serum calcium.
c. Parathormone hormone.
d. Plasma chloride.
e. Serum phosphate.
99.You have a patient who has pheochromocytoma and his blood pressure is
200/120 mmHg. You are preparing this patient for surgery. Which of the
following drugs should be used FIRST for the control of blood pressure?
a. Phenoxybenzamine.
b. Propranolol.
c. Nifedipine.
d. Atenolol.
e. Captopril.
2. A 30-year-old male received a blow to the front of his arm. Following that he
suffered from weakness in flexion and supination of the forearm. Which of the
following nerves is MOST likely to be injured?
a. Axillary.
b. Median.
c. Musculocutaneous.
d. Radial.
e. Ulnar.
5. A 74-year-old woman present to the outpatient clinic with pelvic pain. A CT scan
reveals enlarged para-aortic lymph nodes. These are MOST likely to be involved
in secondary spread from a tumour in which of the following organs?
a. Cervical.
b. Ovary.
c. Uterus.
d. Vagina.
e. Vulva.
P a g e 2 | 25
ElEMAM’s SurgiNotes 2022
9. A CT scan of the a 63-year-old man reveals that the left renal vein is occluded as
it crosses the aorta. The occlusion is due to compression by an arterial aneurism
anterior to the vein. What is the MOST likely location of aneurism?
a. Coeliac artery.
b. Inferior mesenteric artery.
c. Left colic artery.
d. Middle colic artery.
e. Superior mesenteric.
11.A 35-year-old woman is seen at the surgical clinic with a history of faecal
incontinence since the birth of her second child 18 months previously. The first
stage of labour had been prolonged and difficult. Physical examination reveals
a relatively lax anal sphincter. Which nerve is likely to have been damaged in
labour?
a. Genitofemoral nerve.
b. Lumbosacral trunk.
c. Obturator nerve.
d. Pelvic splanchnic nerve.
e. Pudendal nerve.
12.A 65-year-old man presents with hematuria and right lion pain. Computerized
tomography demonstrates a right renal tumour and he undergoes a right radical
nephrectomy. The right testicular vein drains into which of the following?
a. Inferior mesenteric vein.
b. Inferior vena cava.
c. Right adrenal vein.
d. Right lumber vein.
e. Right renal vein.
P a g e 3 | 25
ElEMAM’s SurgiNotes 2022
13. A 19-year-old man is brought to the emergency department after dislocation his
shoulder while playing football. Following treatment of the dislocation. He cannot
initiate abduction of his arm. An MRI of the shoulder shows a torn muscle. Which
muscle was MOST likely damaged by the injury?
a. Coracobrachialis.
b. Long head of the triceps.
c. Pectoralis minor.
d. Supraspinatus.
e. Teres major.
14. A 32-years-old male patient is admitted to the emergency department with groin
pain. Examination reveals that the patient has an indirect inguinal hernia. Which of
the following nerves is compressed by the herniated structure In the inguinal canal
to give the patient pain?
a. Iliohypogastric.
b. Lateral femoral cutaneous.
c. Ilioinguinal.
d. Subcostal.
e. Pudendal.
15. A 44-year-old man is admitted to the emergency department with excessive
vomiting and dehydration. Radiographic images demonstrate that part of bowel is
being compressed between the abdominal aorta and the superior mesenteric
artery. Which of the following intestinal structures is MOST likely being
compressed?
a. second part of duodenum.
b. Transverse colon.
c. Third part of duodenum.
d. First part of duodenum.
e. Jejunum.
16. A 34-year-old is undergoing an emergency appendectomy has been performed
successfully. The patient undergoes an exploratory laparoscopy. Which of the
following anatomic feature are the MOST useful to distinguish the jejunum form
the ilium?
a. Jejunum has thinner walls compared with the ilium.
b. Jejunum has less mesenteric fat compared with ileum.
c. Jejunum has more numerous vascular arcades compared with the ileum.
d. Jejunum has more numerous lymphatic follicles beneath the mucosa
compared with ileum.
e. Jejunum has fewer villi compared with the ileum.
P a g e 4 | 25
ElEMAM’s SurgiNotes 2022
18.Which of the following cell types is essential for normal wound healing?
a. Erythrocytes.
b. Leukocytes.
c. Lymphocytes.
d. Monocytes.
e. Platelets.
22.What is the site of maximum absorption of short chain fatty acids produced by
bacteria?
a. The caecum.
b. The colon.
c. The duodenum.
d. The ileum.
e. The jejunum.
23.Which of the following is the MOST potent stimulus for contraction of the gall
bladder?
a. Acetylcholine.
b. Cholecystokinin.
c. Intravenous hyperalimentation.
d. secretin.
e. Vagal stimulation.
24.The following acid base data: pH: 7.2, PCO2: 20 mmHg, HCO3-: 8 mmol/L, base
excess: 19 mmol/L, would be MOST consistent with:
a. Anxiety.
b. Lobar collapse of the lung.
c. Pyloric obstruction.
d. Septic shock.
e. Starvation.
P a g e 6 | 25
ElEMAM’s SurgiNotes 2022
27.A 21-year-old graduate student has a large hypertrophic scar on the lower part
of face. The patient has sustained a laceration on her face 2 years previously
after injuring her face. Which of the following statement regarding scar revision
is TRUE?
a. scar maturation refers to the change in size of the wound in the first 1 to
2 months.
b. Scar revision should have been performed in the first 3 months after
injury to minimize fibrosis.
c. Revision should be performed earlier in children than in adults.
d. It corrects undesirable pigmentation.
e. Scar revision should be delayed approximately 1 year to allow
maturation.
41. Bacteroides fragilis responds BEST to adequate doses of which of the following
antibiotics?
a. Cefoxitin.
b. Clindamycin.
c. Erythromycin.
d. Tazocin.
e. Tobramycin.
42. A 30-year-old male patient was involved in a car accident. He had severe lacerated
wound involving the gluteal and thigh regions with laceration of bulky muscles.
What is the MOST important measure in the management of this patient?
a. Hyperbaric O2.
b. Insertion of a drain.
c. Local antibiotics powder.
d. Removal of foreign and dead tissues.
e. Skin graft.
43. Which of the following cells will be involved in the chronic inflammatory reaction
which follows the introduction of foreign body?
a. Eosinophils.
b. Giant cells.
c. Mast cells.
d. Neutrophils.
e. Plasma cells.
44. A patient with a non-obstructing carcinoma of the colon is being prepared for
elective resection. Which of the following reduces the risk of postoperative
infectious complication?
a. Avoidance of oral antibiotics to prevent emergence of Clostridium difficile.
b. Postoperative administration for 48 h of parenteral antibiotics effective
against aerobes and anaerobes.
c. Postoperative administration of parenteral antibiotics effective against
aerobes and anaerobes until the patient's intravenous lines and all other
drains are removed.
d. Redosing of antibiotics in the operating room if the case lasts for more than
2 hours of operating time.
e. Single preoperative parenteral dose of antibiotic effective against aerobes.
P a g e 11 | 25
ElEMAM’s SurgiNotes 2022
47.Which of the following has the GREATEST influence for impaired healing of a
wound?
a. Anaemia.
b. Diabetes mellitus.
c. Intake of corticosteroids.
d. Local wound infection.
e. Malnutrition.
55.An 80-year-old woman who lives in a nursing home and who had just finished a
10-days course of antibiotics. Has abdominal pain and profuse diarrhea. Her
stool is tested and comes back positive for Cl. difficile. What is the MOST
appropriate initial management?
a. Oral vancomycin.
b. Intravenous vancomycin.
c. Metronidazole.
d. Vancomycin enemas.
e. Supportive treatment only.
56.Which of the following statement about the usage of antibiotics for the
prevention of surgical site infection is NOT TRUE?
a. Antibiotics should be administrated within 60 minutes of incision.
b. The therapeutic dose of the antibiotic should be administrated
intravenously.
c. Adequate tissue concentrations should be maintained during operation
by re-dosing as necessary.
d. Antibiotics should be continued for 48 hours after operation.
e. Vancomycin is the agent of chronic for patients from nursing homes
undergoing hip replacement.
P a g e 14 | 25
ElEMAM’s SurgiNotes 2022
61. What is the MOST common cause of death 2-years after heart transplantation?
a. Accelerated graft arteriosclerosis.
b. Arrhythmia.
c. Cardiomyopathy.
d. Infection.
e. Malignancy.
65.The arterial blood gas of a patient was pH 7.5, PCO2 47 mmHg, HCO3- 35 mmol/L.
This patient MOST likely is suffering from:
a. Chronic obstructive pulmonary disease.
b. Diabetic ketoacidosis.
c. Persistent diarrhea.
d. Profound vomiting.
e. Salicylate poisoning.
69.A 24-year-old woman has acute renal failure following postpartum hemorrhage.
Laboratory studies showed serum glucose 150 mg/dl, sodium 135mEq/L,
potassium 6.5 mEq/L, chloride 105 mEq/L and bicarbonate 15 mEq/L. Which of
the following is recommended?
a. Decrease potassium chloride to 10 mEq/L.
b. Intravenous 0.9% sodium chloride.
c. 100 ml of 50% glucose water with 10 U insulin.
d. Intravenous calcitonin.
e. Intravenous magnesium sulfate.
71.A 50-year-old man presented with sever repeated vomiting as a result of gastric
outlet obstruction is admitted to the hospital. There is marked dehydration,
with urine output 20 ml/h and the hematocrit is 48%. Initial treatment for this
patient should include which of the following?
a. Administration of 10% dextrose (D10W) in one-third saline solution IV.
b. Antiemetics.
c. Ringer's Lactate solutions.
d. Hemodialysis to correct azotemia.
e. Saline fluid replacement with appropriate potassium administration.
75.The anion gap will increase with an increase in the plasma concentration of:
a. Sodium.
b. Potassium.
c. Chloride.
d. Bicarbonate.
e. Lactate.
78.A patient with a life threatening pulmonary embolus is receiving heparin. She
developed serious vaginal bleeding and a major retroperitoneal haematoma
after 5 days of heparin therapy. What is the recommended treatment?
a. Reverse heparin and evacuate the hematoma.
b. Reverse heparin by protamine sulphate and insert a vena caval filter.
c. stop heparin and closely observe the patient.
d. stop heparin, give fresh frozen plasma and start warfarin therapy.
e. Switch to low-dose heparin.
81.What does increased level of fibrin degradation products (FDP) in the blood
denote?
a. Diminished fibrin synthesis.
b. Diminished platelet count.
c. Diminished platelet function.
d. Disseminated intravascular coagulation (DIC).
e. Heparin overdose.
85.A patient who received 8 units of blood during an operation developed diffuse
oozing of blood at the site of the wound. Which of the following tests will be
helpful for the diagnosis of the etiology of this ooze?
a. Platelet count.
b. Bone marrow biopsy.
c. Liver-spleen scan.
d. Factor VIII assay.
e. Smear for Howell-Jolly bodies.
86.A 5-year-old boy slipped and hurted his right knee while walking. He presents
with a tender, swollen, warm knee with significant hemarthrosis. His PT is 12
(Normal, 13 second), PTT is over 100 (Normal, 25 second), platelet count is
300,000/uL and bleeding time is normal. Initial management should consist of
which of the following?
a. Fresh frozen plasma.
b. Aspiration of knee.
c. Factor VIII concentrate.
d. Passive exercise.
e. Long-leg cast.
87.Which of the following denotes a hemolytic transfusion reaction during
anesthesia?
a. Shaking, chills and muscle spasms.
b. Fever and oliguria.
c. Hyperpyrexia and hypotension.
d. Tachycardia.
e. Bleeding and hypotension.
88.A 22-year-old man is brought into the emergency department in profound shock
after a full from the fourth floor of a building. After resuscitation, small bowel
resection and hepatic segmentectomy are performed at laparotomy. He
receives 15 units of packed RBCs, 4 Units of fresh-frozen plasma and 8 L of
Ringer's lactate. On closure, diffuse oozing of blood is noted. What is the MOST
likely cause?
a. Hepatic failure.
b. Hypersplenism.
c. Platelet deficiency.
d. Factor IX (Christmas factor) deficiency.
e. Congenital hypoprothrombinemia.
P a g e 22 | 25
ElEMAM’s SurgiNotes 2022
92.Which of the following is the MOST potent stimulus for aldosterone secretion?
a. ACTH.
b. Antidiuretic hormone.
c. Hyperkalemia.
d. Hypernatremia.
e. The renin angiotensin system.
P a g e 23 | 25
ElEMAM’s SurgiNotes 2022
94.You have a patient who has pheochromocytoma and his blood pressure is
200/120 mmHg. You are preparing this patient for surgery. What is the following
drugs should be FIRST for the control of blood pressure?
a. Phenoxybenzamine.
b. Propranolol.
c. Nifedipine.
d. Atenolol.
e. Captopril.
97.A 40-year-old women had the anterior lobe of the pituitary removed because of
a tumour. Without postoperative supplements, which of the following could
occur?
a. Failure to produce adequate amounts of thyroxine.
b. Failure to produce parathyroid hormone in response to hypocalcemia.
c. Failure to secrete catecholamines in response to stress.
d. Failure to secrete insulin in hyperglycaemia.
e. Inability to concentrate urine in response to water deprivation.
99.A 50-year-old woman present with lethargy, weight gain, cold intolerance and
loss of interest for the past six months. Which is the MOST appropriate initial
investigation?
a. Erythrocyte sedimentation rate estimation.
b. Radioactive isotope scan of thyroid.
c. Thyroid antibodies screen.
d. Thyroid stimulating hormone estimation.
e. Ultrasound scan of thyroid gland.
100. A 48-year-old woman with breast cancer diagnosis five years ago is admitted
with a two week history of back pain, nausea, fatigue and constipation. Her
serum calcium is 14 mg/dl. Which of the following is the CORRECT first line
treatment for this patient?
a. Bisphosphonate infusion.
b. Calcitonin infusion.
c. Dextrose/insulin infusion.
d. Forced diuresis with furosemide.
e. Intravenous fluids.
P a g e 25 | 25
ElEMAM’s SurgiNotes 2022
5. A 65-year-old man presents with haematuria and left loin pain. Computerized
tomography confirms a left renal tumour. The normal left renal hilum lies at
which vertebral level?
a. T11.
b. T12.
c. L1.
d. L2.
e. L3.
P a g e 2 | 25
ElEMAM’s SurgiNotes 2022
11.A CT scan of a 63-year-old man reveals that the left renal vein is occluded as it
crosses the aorta. The occlusion is due to compression by an arterial aneurysm
anterior to the vein. What is the MOST likely location of this aneurysm?
a. Coeliac artery.
b. Inferior mesenteric artery.
c. Left colic artery.
d. Middle colic artery.
e. Superior mesenteric artery.
14.A 22-year-old man is admitted to the emergency department after falling from
his bicycle. Radiograph examination reveals a fracture of the tibia above ankle.
MRI and physical examination reveal the tibial nerve is severed on the posterior
aspect of tibia. Which of the following signs will MOST likely present during
physical examination?
a. Sensory loss of the dorsum of the foot.
b. Sensory loss on the sole of the foot.
c. Foot drop.
d. Paralysis of the extensor digitorum brevis.
e. Sensory loss of the entire foot.
P a g e 4 | 25
ElEMAM’s SurgiNotes 2022
23.A fit 30-year-old man donates 500 ml of blood. Which one of the following is the
MOST likely physiological change?
a. A fall in blood pressure.
b. Activation of renin-angiotensin system.
c. Reduced urine output.
d. Sweating.
e. Tachypnoea.
25. During the second (Proliferative) phase of wound healing the predominant cells in
the wound site are fibroblasts. This cell is of mesenchymal origin and produces the
matrix and collagen needed to strengthen the scar. Cross linkage of collagen
requires hydroxy-proline and hydroxy-lysine residues, which requires a specific
vitamin to be available in sufficient quantities. Deficiency of which vitamin results
in collagen that is unstable?
a. Vitamin B2 (Riboflavin).
b. Vitamin B6 (Pyridoxine).
c. Vitamin C (Ascorbic acid).
d. Vitamin D (Cholecalciferol).
e. Vitamin E (Tocopherol).
26. A 78-year-old woman with emphysema receiving 28% oxygen by mask has the
following blood gas results:
pH PO2 PaCO2 Bicarbonate Base excess
Finding: 7.28 70 mm Hg 48 mm Hg 36 mmol/L +5
The MOST likely interpretation is:
a. Mixed respiratory and metabolic acidosis.
b. Partially compensated metabolic acidosis.
c. Partially compensated respiratory acidosis.
d. Uncompensated metabolic acidosis.
e. Uncompensated respiratory acidosis.
27. A 70-year-old patient with diabetes and paraplegia is undergoing an elective
laparoscopic cholecystectomy after an episode of biliary pancreatitis. Shortly after
induction, blood pressure is normal, but ECG shows peaked P waves and a widened
QRS complex. The MOST likely diagnosis is:
a. Ketoacidosis.
b. Hyperkalemia.
c. Hypoglycemia.
d. Hypocalcemia.
e. Acute myocardial infarction.
28. A 70-year-old man with chronic obstructive pulmonary disease is admitted for
elective hemicolectomy. What is a preoperative arterial blood gas analysis likely to
show?
Arterial PCO2 Bicarbonate.
a. Decreased Decreased.
b. Decreased Increased.
c. Decreased Normal.
d. Increased Decreased.
e. Increased Increased.
P a g e 7 | 25
ElEMAM’s SurgiNotes 2022
29.Which of the following is TRUE regarding the release of tumour necrosis factor-
α?
a. Can be effectively blocked by anti-TNF- α, antibodies to halt systemic
inflammatory response syndrome (SIRS).
b. Does not have any beneficial effects in the early phases of the
inflammatory response.
c. Is primarily from leukocytes.
d. Promotes polymorphonuclear (PMN) cell adherence and further
cytokines release.
e. Is always deleterious.
P a g e 8 | 25
ElEMAM’s SurgiNotes 2022
35.A 45-year-old woman is admitted with a chronic cough and weight loss. On
examination, crepitations are heard over the apex of the right lung. A chest X-
ray reveals cavitation in the right apex. What is a lung biopsy MOST likely to
show?
a. Abundant plasma cells.
b. Aggregates of activated macrophages.
c. Eosinophils.
d. Mast cells.
e. Neutrophilic infiltrates with abscess formation.
37.In the presence of acute inflammation, when an abscess has formed, which of
the following cell types is MOST likely to be primarily involved in phagocytic
activity?
a. Erythrocytes.
b. Lymphocytes.
c. Macrophages.
d. Mast cells.
e. Plasma cells.
39.Which of the following is the MOST likely cause of pyrexia occurring 48 hours
after an abdominal operation?
a. Chest infection.
b. DVT.
c. Leaking intestinal anastomosis.
d. Pulmonary embolism.
e. Wound infection.
43.A 45-year-old diabetic male patient complains of severe pain in the thigh.
Examination reveals spreading oedema, swelling and tenderness of the thigh. A
provisional diagnosis of necrotizing fasciitis is made. Which of the following is
MOST important in the treatment?
a. Antifungal agents.
b. Antitoxin.
c. Hyperbaric O2.
d. Immunoglobulins.
e. Wide surgical debridement.
50.A 30-year-old woman sustained a puncture wound to the foot. The patient has
been on a therapeutic dose of steroids for the past 5 years for ulcerative colitis.
Her last tetanus toxoid booster was 8 years ago. What should the patient
receive?
a. Tetanus toxoid booster.
b. Human immunoglobulin, antibiotics with anaerobic coverage.
c. Tetanus toxoid plus human immunoglobulin.
d. Tetanus toxoid plus human immunoglobulin and antibiotics with aerobic
and anaerobic coverage.
e. Wide debridement of the wound.
52.An 80-year-old woman who lives in a nursing home and who had just finished a
10-day course of antibiotics has abdominal pain and profuse diarrhea. Her stool
is tested and comes back positive for Cl. difficile. What is the MOST appropriate
initial management?
a. Oral vancomycin.
b. Intravenous vancomycin.
c. Metronidazole.
d. Vancomycin enemas.
e. Supportive treatment only.
P a g e 13 | 25
ElEMAM’s SurgiNotes 2022
57.A patient is reported to have a potassium level of 7.3 mEq/L. What is the FIRST
diagnostic or therapeutic maneuver for this patient?
a. An ECG and administration of 1 g of 10% calcium gluconate.
b. An ECG and infusion of 45 mEq/L NaHCO3.
c. An ECG and infusion of glucose and insulin intravenously.
d. Kayexalate enema.
e. Transfer to an intensive care unit for careful cardiac monitoring.
58.The arterial blood gas analysis of a patient was pH 7.5, P(CO2) 47 mmHg, HCO3-
35 mmol/L. This patient MOST likely is suffering from:
a. Chronic obstructive pulmonary disease.
b. Diabetic ketoacidosis.
c. Persistent diarrhea.
d. Profound vomiting.
e. Salicylate poisoning.
61.A 45-year-old male with a known history of alcoholism is admitted with acute
pancreatitis. His serum calcium is 7 mg/dl. Management is based upon which of
the following?
a. One-fourth of calcium in scrum is ionized.
b. Alkalosis increases the ionized calcium component.
c. Hypocalcemia may cause polyuria and polydipsia.
d. Determination of serum albumin is necessary for proper determination of
serum calcium level.
e. 20% of serum calcium is bound to citrates.
64.Features of adult respiratory distress syndrome (ARDS) include all the following
EXCEPT:
a. Increased lung compliance.
b. Diffuse pulmonary infiltrates on chest X ray.
c. Increase alveolar arterial oxygen gradient.
d. Diuretics may help in recovery of these patients.
e. Artificial ventilation when required must be based on pressure based
ventilation.
P a g e 16 | 25
ElEMAM’s SurgiNotes 2022
65.A 40-year-old man is confused and restless the second day after upper
abdominal surgery and repair of a hiatus hernia. What is the MOST probable
cause of his condition?
a. Pulmonary embolism.
b. Narcotic overdose.
c. Pulmonary atelectasis.
d. Electrolyte imbalance.
e. Starvation ketosis.
67.A 39-year-old woman is making a slow but adequate recovery after sustaining a
40% surface area burn injury. On the sixth postoperative day she becomes
unwell. She vomits intermittently, has painless abdominal distension and starts
to hiccup. What is the MOST likely cause of these symptoms?
a. Acute gastric dilatation.
b. Acute intestinal obstruction.
c. Clostridium difficile infection.
d. Faecal impaction.
e. Systemic sepsis.
69.Which of the following types of shock is associated with high pulmonary wedge
pressure?
a. Hypovolemic shock.
b. Cardiogenic shock.
c. Early septic shock.
d. Late septic shock.
e. Neurogenic shock.
74.A 55-year-old male patient is receiving 150 mg of aspirin daily because he has
coronary artery disease. Which of the following tests will be affected in this
patient?
a. Bleeding time.
b. Coagulation time.
c. INR.
d. PTT.
e. Thrombin time.
76.A 30-year-old male patient was admitted to the casualty department due to a
car accident. The patient had fracture of the pelvis and the right femur and he
received 5 liters of blood following which he started to have bleeding from his
nose and mouth. What is the MAIN cause of this bleeding tendency?
a. Decrease in fibrinogen.
b. Decrease in prothrombin.
c. Decrease of calcium.
d. Increased fibrinolytic activity.
e. Platelet depletion.
P a g e 19 | 25
ElEMAM’s SurgiNotes 2022
79.You were obliged to operate for a strangulated inguinal hernia in a patient who
was receiving 300 mg aspirin daily. During surgery there was excessive bleeding.
What would you advise?
a. Desmopressin.
b. Fresh blood.
c. Fresh frozen plasma.
d. Fresh platelets.
e. IV vitamin K.
83.A 5-year-old boy slipped and hurt his right knee while walking. He presents with
a tender, swollen, warm knee with significant hemarthrosis. His PT is 12
(Normal, 13 seconds), PTT is over 100 (Normal, 25 seconds), platelet count is
300,000/mm3 and bleeding time is normal. Initial management should consist
of which of the following?
a. Fresh-frozen plasma.
b. Aspiration of knee.
c. Factor VIII concentrate.
d. Passive exercise.
e. Long-leg cast.
85.A 50-year-old female patient has chronic renal failure and has been maintained
on chronic dialysis. The patient underwent cholecystectomy. Post-operatively
she had severe bleeding. What is the MOST likely cause for this bleeding?
a. Elevated PT.
b. Elevated PTT.
c. Low platelet count.
d. Decreased platelet aggregation.
e. Sepsis.
86.A 70-year-old female patient is receiving warfarin because she has AF and had
previous thrombosis. Which of the following statements regarding warfarin is
CORRECT?
a. The dose of warfarin is adjusted according to the partial thromboplastin
time.
b. Warfarin takes about 8 hours to exert its effects.
c. Warfarin acts by inhibiting factor XII.
d. If the patient is also taking aspirin, the dose of warfarin should be
reduced.
e. Protamine sulphate is the antidote to warfarin
P a g e 22 | 25
ElEMAM’s SurgiNotes 2022
90.What is the hormone that acts on the intestines to increase calcium absorption?
a. Calcitonin.
b. Corticotrophin releasing factor (CRF).
c. Pancreatic polypeptide.
d. Parathormone.
e. Thyroxine.
91.A 30-year-old female patient complains of fatigue, generalized bony aches and
depression. The patient gives a history of 2 previous operations for urinary
stones. Which of the following laboratory tests is MOST accurate in the
diagnosis?
a. 24 hours urinary calcium.
b. Ionizable serum calcium.
c. Total serum calcium .
d. Plasma chloride.
e. Serum phosphate.
96.A 53-year-old man with renal failure presents with nausea, headache and
pruritus. Blood tests show a serum calcium of 13 mg/dl (Normal 9-10.5 mg/dl).
Which of the following is the MOST likely diagnosis?
a. Hypoparathyroidism.
b. Primary hyperparathyroidism.
c. Pseudohypoparathyroidism.
d. Secondary hyperparathyroidism.
e. Tertiary hyperparathyroidism.
P a g e 24 | 25
ElEMAM’s SurgiNotes 2022
2. Taste from the posterior one third of tongue is provided by which of the
following nerves?
a. Facial.
b. Glossopharyngeal.
c. Hypoglossal.
d. Lingual.
e. Vagus.
10.A 34-year-old man is seen in the emergency department with a fibular fracture
following a football match. On examination he is noted to have loss of foot
eversion. Which area of skin should be examined to confirm loss of the
cutaneous distribution of the affected nerve?
a. Along the lateral aspect of the foot.
b. Along the medial aspect of the foot.
c. Between the hallux and the second digit.
d. On the dorsal surface of the foot.
e. On the plantar surface of the foot.
11.Which of the following is the sensory supply to the skin of the tip of the index
finger?
a. Radial nerve only.
b. Radial and median nerves.
c. Median and ulnar nerves.
d. Ulnar nerve only.
e. Median nerve only.
12.Which one of the following structures provides the GREATEST support for the
liver?
a. The falciform ligament.
b. The coronary ligament.
c. The ligamentum teres.
d. The hepatic veins joining the inferior vena cava.
e. The ligamentum venosum.
P a g e 3 | 25
ElEMAM’s SurgiNotes 2022
13.A 65-year-old man presents with haematuria and right loin pain. Computerized
tomography demonstrates a right renal tumour and he undergoes a right radical
nephrectomy. The right testicular vein drains into which of the following?
a. Inferior mesenteric vein.
b. Inferior vena cava.
c. Right adrenal vein.
d. Right lumbar vein.
e. Right renal vein.
14.A 62-year-old man is admitted to the emergency department due to retention
of urine. Physical examination reveals prostatic hypertrophy. After several
unsuccessful attempts to catheterize the penile urethra, the urologist orders
drainage of the urinary bladder by the least invasive procedure, avoiding entry
into the peritoneal cavity or the injury of any major vessels or organs. Which of
the following spaces needs to be traversed by the needle to reach the bladder?
a. Ischioanal fossa.
b. Perineal body.
c. Retropubic space (Of Retzius).
d. Superficial perineal cleft.
e. Deep perineal pouch.
15.A 20-year-old male while walking slipped on the wet pavement and injured his
right arm. Radiographic images showed a midshaft fracture of the humerus.
Which pair of structures was MOST likely injured at the fracture site?
a. Median nerve and brachial artery.
b. Axillary nerve and posterior humeral circumflex artery.
c. Radial nerve and deep brachial artery.
d. Suprascapular nerve and artery.
e. Long thoracic nerve and lateral thoracic artery.
16.A 29-year-old female is examined in the emergency department after falling
from her balcony. Radiographic examination reveals that she has suffered a
broken clavicle, with associated internal bleeding. Which of the following
vessels is MOST likely to be injured in clavicular fractures?
a. Subclavian artery.
b. Cephalic vein.
c. Lateral thoracic artery.
d. Subclavian vein.
e. Internal thoracic artery.
P a g e 4 | 25
ElEMAM’s SurgiNotes 2022
17.Which of the following tests will be affected after intake of non-steroidal anti-
inflammatory drugs?
a. Bleeding time.
b. Coagulation time.
c. INR.
d. PTT.
e. Thrombin time.
P a g e 5 | 25
ElEMAM’s SurgiNotes 2022
25.A 60-year-old woman had a total pancreatectomy three months ago. She now
complains of offensive, foul smelling stools that are difficult to flush. Lack of
which enzyme is responsible?
a. Amylase.
b. Cholecystokinin.
c. Lipase.
d. Secretin.
e. Trypsin.
28.Pancreatic acini secrete enzyme that play an important role in the digestion of
proteins. What is the MOST potent stimulant of pancreatic acinar cells?
a. Acetylcholine.
b. Cholecystokinin (CCK).
c. Peptide.
d. Secretin.
e. Trypsinogen.
P a g e 7 | 25
ElEMAM’s SurgiNotes 2022
31.Which of the following is the MOST potent stimulus for contraction of the gall
bladder?
a. Acetylcholine.
b. Cholecystokinin.
c. Intravenous hyperalimentation.
d. Secretin.
e. Vagal stimulation.
33.A 40-year-old male patient was injured in the thigh and a bleeding point was
ligated by a silk suture. The patient got recurrent attacks of inflammation at the
site of the wound. The surgeon explored the wound and removed an
inflammatory reaction mass around the silk suture. Which of the following cells
will be the predominant cell detected in this mass?
a. Basophils.
b. Eosinophils.
c. Lymphocyte.
d. Macrophages.
e. Neutrophils.
37.A 45-year-ol woman is admitted with a chronic cough and weight loss. On
examination, crepitations are heard over the apex of the right lung. A chest X-
ray reveals cavitation in the right apex. What is a lung biopsy MOST likely to
show?
a. Abundant plasma cells.
b. Aggregates of activated macrophages.
c. Eosinophils.
d. Mast cells.
e. Neutrophilic infiltrates with abscess formation.
42.Which of the following has the GREATEST influence for impaired healing of a
wound?
a. Anaemia.
b. Diabetes mellitus.
c. Intake of corticosteroids.
d. Local wound infection.
e. Malnutrition.
43.A 30-year-old woman sustained a puncture wound to the foot. The patient has
been on a therapeutic dose of steroids for the past 5 years for ulcerative colitis.
Her last tetanus toxoid booster was 8 years ago. What should the patient
receive?
a. Tetanus toxoid booster.
b. Human immunoglobulin, antibiotics with anaerobic coverage.
c. Tetanus toxoid plus human immunoglobulin.
d. Tetanus toxoid plus human immunoglobulin and antibiotics with aerobic
and anaerobic coverage.
e. Wide debridement of the wound.
44.The clinical course of the majority of patients with HCV infection is characterized
by which one of the following?
a. Acute constitutional symptoms and jaundice.
b. Acute fulminant hepatic failure.
c. Development of chronic hepatitis.
d. Progression to cirrhosis.
e. Development of hepatocellular carcinoma.
P a g e 11 | 25
ElEMAM’s SurgiNotes 2022
46.Which of the following statements about the usage of antibiotics for the
prevention of surgical site infection is NOT TRUE?
a. Antibiotics should be administered within 60 minutes of incision.
b. The therapeutic dose of the antibiotic should be administered
intravenously.
c. Adequate tissue concentrations should be maintained during operation
by re-dosing as necessary.
d. Antibiotics should be continued for 48 hours after operation.
e. Vancomycin is the agent of choice for patients from nursing homes
undergoing hip replacement.
48.A 24-year-old HIV positive man presents to the emergency room with acute
onset pain and redness in his scrotum, penis and perineum. Upon examination,
you feel crepitus over the erythematous area described, which emits a foul-
smelling grey discharge. What is the MOST appropriate management for this
patient's illness?
a. Apply topical polymycin ointment.
b. Give hydrocortisone infusion.
c. Initiate highly active antiretroviral treatment.
d. Initiate penicillin G infusion.
e. Surgical debridement of affected tissue.
P a g e 12 | 25
ElEMAM’s SurgiNotes 2022
56.A 45-year-old male patient had renal transplantation operation. Thirty minutes
after the operation the urinary output markedly dropped. The blood pressure
of was 120/80 mmHg. Duplex scanning of the transplanted kidney revealed
patent renal vessels and normal pelvicalyceal system. Which of the following is
TRUE regarding the present problem?
a. It is less liable to occur in females who had repeated pregnancies.
b. It is more liable to occur in patients who receive liver transplantation.
c. Stimulated CD4 lymphocytes are mainly responsible for the problem.
d. The patient needs immediate removal of the transplanted kidney.
e. The problem can be corrected by increasing the dose of cyclosporin.
P a g e 14 | 25
ElEMAM’s SurgiNotes 2022
61.A patient is reported to have a potassium level of 7.3 mEq/L. What is the FIRST
diagnostic or therapeutic maneuver for this patient?
a. An ECG and administration of 1 g of 10% calcium gluconate.
b. An ECG-and infusion of 45 mEq/L NaHCO3.
c. An ECG and infusion of glucose and insulin intravenously.
d. Kayexalate enema.
e. Transfer to an intensive care unit for careful cardiac monitoring.
62.The arterial blood gas analysis of a patient was pH 7.5, P(CO2) 47 mmHg, HCO3-
35 mmol/L. This patient MOST likely is suffering from:
a. Chronic obstructive pulmonary disease.
b. Diabetic ketoacidosis.
c. Persistent diarrhea.
d. Profound vomiting.
e. Salicylate poisoning.
66.In the presence of acute blood loss, what is the initial mechanism to maintain
an adequate preload to the heart?
a. Development of tachycardia.
b. Hormonal effects of angiotensin.
c. Hormonal effects of rennin.
d. Increase in systemic vascular resistance.
e. Increased cortisol secretion.
73. A 38-year-old man in end-stage renal failure resulting from polycystic kidney disease
receives a cadaveric renal transplant. Good renal function is established but four
weeks later deteriorates, the serum creatinine rising by 25%. Which of the following
processes is MOST likely to be responsible for this deterioration?
a. B-cell mediated rejection.
b. Circulating immune complex disease.
c. IgG antibody mediated rejection.
d. Post-transplant lymphoproliferative disorder.
e. T-cell mediated rejection.
74. A 78-year-old woman with emphysema receiving 28% oxygen by mask has the
following blood gas results:
pH PO2 PaCO2 Bicarbonate Base excess
Finding: 7.28 70 mmHg 48 mmHg 36 mmol/L +5
Normal: 7.35-7.45 90-110 35-45 22-26 -2 to +2
The MOST likely interpretation is:
a. Mixed respiratory and metabolic acidosis.
b. Partially compensated metabolic acidosis.
c. Partially compensated respiratory acidosis.
d. Uncompensated metabolic acidosis.
e. Uncompensated respiratory acidosis.
75. A 28-year-old motorcyclist is admitted following a road traffic accident, having
sustained bilateral femoral fractures and a ruptured spleen. Three days
postoperatively he is noted to be agitated, hypoxaemic and difficult to ventilate. His
observations show a blood pressure of 120/80 mmHg, regular pulse of 88
beats/minute and he is apyrexial. A chest X-ray shows bilateral diffuse lung infiltrates.
What is the MOST likely underlying diagnosis?
a. Adult respiratory distress syndrome (ARDS).
b. Atelectasis.
c. Bronchopneumonia.
d. Pulmonary oedema.
e. Pulmonary thrombo-embolism.
76. A 22-year-old man involved in a motor vehicle accident is found to have a thoracic
spine fracture (T6) and paraplegia. The patient is hypotensive with systolic BP of 70
mmHg, is bradycardiac with a pulse of 48 beats/min and is breathing comfortably.
Which of the following would the MOST appropriate initial treatment?
a. Isotonic fluid administration.
b. Steroid administration within 24 hours of the injury.
c. Immediate intubation.
d. Alpha-agonist administration.
e. Immediate magnetic resonance imaging.
P a g e 19 | 25
ElEMAM’s SurgiNotes 2022
77.What is the primary source of calories during prolonged starvation (>5 days
fasting)?
a. Skeletal muscle proteins.
b. Body stored fat.
c. Liver glycogen.
d. Ketone bodies.
e. Muscle glycogen.
78.A 40-year-old man is confused and restless the second day after upper
abdominal surgery and repair of a hiatus hernia. What is the MOST probable
cause of his condition?
a. Pulmonary embolism.
b. Narcotic overdose.
c. Pulmonary atelectasis.
d. Electrolyte imbalance.
e. Starvation ketosis.
81.A 30-year-old male patient was admitted to the casualty department due to a
car accident. The patient had fracture of the pelvis and the right femur and he
received 5 liters of blood following which he started to have bleeding from is
nose and mouth. What is the MAIN cause of this bleeding tendency?
a. Decrease in fibrinogen.
b. Decrease in prothrombin.
c. Decrease of calcium.
d. Increased fibrinolytic activity.
e. Platelet depletion.
90.What is the hormone that acts on the intestines to increase calcium absorption?
a. Calcitonin.
b. Corticotrophin releasing factor (CRF).
c. Pancreatic polypeptide.
d. Parathormone.
e. Thyroxine.
91.A 30-year-old female patient complains of fatigue, generalized bony aches and
depression. The patient gives a history of 2 previous operations for urinary
stones. Which of the following laboratory tests is MOST accurate in the
diagnosis?
a. 24 hours urinary calcium.
b. Ionizable serum calcium.
c. Total serum calcium.
d. Plasma chloride.
e. Serum phosphate.
94.A 55-year-old female patient who has metastatic breast cancer presents with
weakness, anorexia, malaise, constipation and back pain and lethargy.
Laboratory studies include a normal chest X-ray; serum albumin 3.2 mg/dl;
serum calcium 14 mg/dl; serum phosphorus 2.6 mg/dl; serum chloride 108
mg/dl; BUN 32 mg/dl and creatinine 2.0 mg/dl. What is the appropriate initial
management?
a. Intravenous normal saline infusion.
b. Administration of thiazide diuretics.
c. Administration of intravenous phosphorus.
d. Use of mithramycin.
e. Neck exploration and parathyroidectomy.
97. Which of the following which is the MOST effective way in preventing surgical
infection?
a. Antibiotic prophylaxis.
b. Bowel preparation.
c. Drains and irrigation.
d. Skin preparation.
e. Surgical technique.
99. A 22-year-old man involved in a motor vehicle accident is found to have a thoracic
spine fracture (T6) and paraplegia. The patient is hypotensive with a systolic BP of
70 mmHg, is bradycardiac with a pulse of 48 beats/min and is breathing
comfortably. Which of the following would the MOST appropriate initial
treatment?
a. Isotonic fluid administration.
b. Steroid administration within 24 hours of the injury.
c. Immediate intubation.
d. Alpha-agonist administration.
e. Immediate magnetic resonance imaging.
100. A 26 -year-old man with history of chronic duodenal presents with repeated
vomiting for 2 days. Insertion of Foley’s catheter in yields 130 ml of concentrated
urine. Serum electrolytes showed sodium 128 mEq/L, potassium 2.9 mEq/L and
chloride 64 mEq/L. Blood pH is 7.53 The MOST appropriate initial management is:
a. 5% dextrose in 0.45 normal saline.
b. 0.45 normal saline plus potassium chloride 60 mEq/L.
c. Lactated Ringer's solution plus potassium chloride 60 mEq/L.
d. Ammonium chloride to correct blood pH.
e. IV furosemide (Lasix) to correct urine output.
P a g e 25 | 25
ElEMAM’s SurgiNotes 2022
13.A 35-year-old woman is seen at the surgical clinic with a history of faecal
incontinence since the birth of her second child 18 months previously. The first
stage of labour had been prolonged and difficult. Physical examination reveals
a relatively lax anal sphincter. Which nerve is likely to have been damaged in
labour?
a. Genitofemoral nerve.
b. Lumbosacral trunk.
c. Obturator nerve.
d. Pelvic splanchnic nerve.
e. Pudendal nerve.
14.A 27-year-old man is admitted to the emergency department after a car crash.
Physical examination reveals weakness in medial rotation and adduction of the
humerus. Which of the following nerves was MOST probably injured?
a. Thoracodorsal.
b. Axillary.
c. Dorsal scapular.
d. Spinal accessory.
e. Radial.
15.A 5-year-old male entered the emergency department with a complaint of
severe abdominal pain. During physical examination it is observed that his
cremasteric reflex is absent. Which of the following nerves is responsible for the
efferent limb of the cremasteric reflex?
a. Ilioinguinal.
b. Iliohypogastric.
c. Genitofemoral.
d. Pudendal.
e. Ventral ramus of T12.
16.A 45-year-old woman is admitted to the emergency department with a
complaint of severe abdominal pain. CT scan and MRI examinations reveal a
tumor of the head of the pancreas involving the uncinate process. Which of the
following vessels is MOST likely to be occluded?
a. Common hepatic artery.
b. Cystic artery and vein.
c. Superior mesenteric artery.
d. Inferior mesenteric artery.
e. Portal vein.
P a g e 4 | 25
ElEMAM’s SurgiNotes 2022
18.A 42-year-old male fell from a height. Radiographic examination reveals fracture
of the proximal femur. Which of the following arteries supplies the proximal part
of the femur?
a. Deep circumflex iliac.
b. Acetabular branch of obturator.
c. Lateral circumflex femoral.
d. A branch of profunda femoris.
e. Medial circumflex femoral.
23.A patient whose blood pH is 7.47, whose PCO2 is 31 mmHg in arterial blood and
whose levels of bicarbonate ion in arterial blood are 23 mEq/liter is in:
a. Compensated metabolic alkalosis.
b. Uncompensated respiratory acidosis.
c. Uncompensated respiratory alkalosis.
d. Uncompensated metabolic acidosis.
e. Uncompensated metabolic alkalosis.
26.A fit 30-year-old man donates 500 ml of blood. Which one of the following is the
MOST likely physiological change?
a. A fall in blood pressure.
b. Activation of renin-angiotensin system.
c. Reduced urine output.
d. Sweating.
e. Tachypnoea.
28.A 78-year-old woman with emphysema receiving 28% oxygen by mask has the
following blood gas results:
pH PO2 PaCO2 Bicarbonate Base excess
Finding: 7.28 70 mmHg 48 mmHg 36 mmol/L +5
The MOST likely interpretation is:
a. Mixed respiratory and metabolic acidosis.
b. Partially compensated metabolic acidosis.
c. Partially compensated respiratory acidosis.
d. Uncompensated metabolic acidosis.
e. Uncompensated respiratory acidosis.
P a g e 7 | 25
ElEMAM’s SurgiNotes 2022
29.A 40-year-old woman had the anterior lobe of the pituitary removed because of
a tumour. Without postoperative supplements, which of the following could
occur?
a. Failure to produce adequate amounts of thyroxine.
b. Failure to produce parathyroid hormone in response to hypocalcaemia.
c. Failure to secrete catecholamines in response to stress.
d. Failure to secrete insulin in hyperglycaemia.
e. Inability to concentrate urine in response to water deprivation.
32.A 70-year-old man with chronic obstructive pulmonary disease is admitted for
elective hemicolectomy. What is a preoperative arterial blood gas analysis likely
to show?
Arterial pCO2 Bicarbonate
a Decrease Decreased
b Decreased Increased
c Decreased Normal
d Increased Decreased
e Increased Increased
P a g e 8 | 25
ElEMAM’s SurgiNotes 2022
34.In patients with reflux esophagitis. What does the presence of columnar cells in
the esophageal mucosa represent?
a. Carcinoma in situ.
b. Carcinoma.
c. Dysplasia.
d. Hyperplasia.
e. Metaplasia.
36.With regard to protein loss after injury, which of the following statements is
TRUE?
a. It can be prevented by total parenteral nutrition.
b. It occurs primarily from skeletal muscle.
c. It occurs primarily from the site of injury.
d. It results from impaired synthesis.
e. It results from significant decrease intake.
P a g e 9 | 25
ElEMAM’s SurgiNotes 2022
37.Which of the following can decrease the adverse effects of steroids on wound
healing?
a. Copper.
b. Vitamin A.
c. Vitamin C.
d. Vitamin D.
e. Vitamin E.
P a g e 10 | 25
ElEMAM’s SurgiNotes 2022
54.What is the MOST common fluid or electrolyte disorder in the surgical patient?
a. Extracellular fluid deficit.
b. Hyperkalemia.
c. Hyponatremia.
d. Metabolic: acidosis.
e. Metabolic alkalosis.
58.A 24-year-old woman has acute renal failure following postpartum hemorrhage.
Laboratory studies showed serum glucose, 150 mg/dl; sodium, 135 mEq/L;
potassium, 6.5 mEq/L; chloride, 105 mEq/L and bicarbonate, 15 mEq/L. Which
of the following is recommended?
a. Decrease potassium chloride to 10 mEq/day.
b. Intravenous 0.9% sodium chloride.
c. 100 ml of 50% glucose water with 10 U insulin.
d. Intravenous calcitonin.
e. Intravenous magnesium sulfate.
59.A 55-year-old man sustains numerous injuries involving the abdomen and lower
extremities. During the intra- and postoperative periods, he is resuscitated with
10 L of Ringer's lactate and 2 U of packed red blood cells (RBC). After initial
improvement, he has severe dyspnea on the second postoperative day. The
MOST useful initial diagnostic test is which of the following?
a. Electrocardiogram.
b. Analysis of arterial blood gases.
c. Insertion of a central venous line.
d. Ventilation-perfusion scan.
e. Computed tomography (CT) scan of abdomen.
62.A 5-year-old man had undergone exploratory laparotomy for perforated gastric
ulcer with severe peritoneal contamination. Six hours after surgery, he is
tachycardic, hypertensive and has shallow respirations. Intubation and
institution of ventilatory support is indicated in the presence of which of the
following?
a. Respiratory rate of 23 breaths/min.
b. PaCO2 of 45 mmHg.
c. PaO2 of 55 mmHg on room air.
d. Heart rate of 140 bpm.
e. BP of 150/100 mmHg.
63.What is the primary source of calories during acute starvation (< 5 days fasting)?
a. Skeletal muscle proteins.
b. Body stored fat.
c. Liver glycogen.
d. Ketone bodies.
e. Muscle glycogen.
64.Which of the following types of shock is associated with high pulmonary wedge
pressure?
a. Hypovolemic shock.
b. Cardiogenic shock.
c. Early septic shock.
d. Late septic shock.
e. Neurogenic shock.
P a g e 16 | 25
ElEMAM’s SurgiNotes 2022
68.A 38-year-old man in end-stage renal failure resulting from polycystic kidney
disease receives a cadaveric renal transplant. Good renal function is established
but four weeks later deteriorates, the serum creatinine rising by 25%. Which of
the following processes is MOST likely to be responsible for this deterioration?
a. B-cell mediated rejection.
b. Circulating immune complex disease.
c. IgG antibody mediated rejection.
d. Post-transplant lymphoproliferative disorder.
e. T-cell mediated rejection.
P a g e 17 | 25
ElEMAM’s SurgiNotes 2022
69.A 55 year-old male patient is receiving 150 mg of aspirin daily because he has
coronary artery disease. Which of the following tests will be affected in this
patient?
a. Bleeding time.
b. Coagulation time.
c. INR.
d. PTT.
e. Thrombin time.
70.A 30 year-old male patient was admitted to the casualty department due to a
car accident. The patient had fracture of the pelvis and the right femur and he
received 5 liters of blood following which he started to have bleeding from his
nose and mouth. What is the MAIN cause of this bleeding tendency?
a. Decrease in fibrinogen.
b. Decrease in prothrombin.
c. Decrease of calcium.
d. Increased fibrinolytic activity.
e. Platelet depletion.
71.A patient with a life threatening pulmonary embolus is receiving heparin. She
developed serious vaginal bleeding and a major retroperitoneal haematoma
after 5 days of heparin therapy. What is the recommended treatment?
a. Reverse heparin and evacuate the haematoma.
b. Reverse heparin by protamine sulphate and insert a vena caval filter.
c. Stop heparin and closely observe the patient.
d. Stop heparin, give fresh frozen plasma and start warfarin therapy.
e. Switch to low-dose heparin.
74.You were obliged to operate for a strangulated inguinal hernia in a patient who
was receiving 300 mg aspirin daily. During surgery there was excessive bleeding.
What would you advise?
a. Desmopressin.
b. Fresh blood.
c. Fresh frozen plasma.
d. Fresh platelets.
e. IV vitamin K.
76.Which of the following is TRUE regarding a male patient who has haemophilia?
a. He has frequent attacks of spontaneous mucocutaneous bleeding.
b. His sisters are affected as well.
c. The partial thromboplastin time is normal.
d. The prothrombin time is prolonged.
e. Transfusion of factor VIII concentrate is helpful.
P a g e 19 | 25
ElEMAM’s SurgiNotes 2022
80.You are planning to perform colectomy for a 55-year-old male patient who had
a previous CABG operation and is receiving 150 mg of aspirin daily. What will be
your policy?
a. Continue aspirin and administer fresh frozen plasma intraoepratively.
b. Continue aspirin and administer fresh platelets intraoperatively.
c. Stop aspirin for 5 days preoperatively and substitute it by warfarin.
d. Stop aspirin for 10 days preoperatively and substitute it by subcutaneous
heparin.
e. Stop aspirin for 3 days preoperatively.
P a g e 20 | 25
ElEMAM’s SurgiNotes 2022
81.Which of the following is the BEST method of identifying the liability to bleeding
during a surgical procedure?
a. Platelet count.
b. A complete history and physical examination.
c. Bleeding time.
d. Lee-White clotting time.
e. Prothrombin time (PT).
84.A 50-year-old female patient bas chronic renal failure and has been maintained
on chronic dialysis. The patient underwent cholecystectomy. Post-operatively
she had severe bleeding. What is the MOST likely cause for this bleeding?
a. Elevated PT.
b. Elevated PTT.
c. Low platelet count.
d. Decreased platelet aggregation.
e. Sepsis.
P a g e 21 | 25
ElEMAM’s SurgiNotes 2022
85.70-year-old female patient is receiving warfarin because she has AF and had
previous thrombosis. Which of the following statements regarding warfarin is
CORRECT?
a. The dose of warfarin is adjusted according to the partial thromboplastin
time.
b. Warfarin takes about 8 hours to exert its effects.
c. Warfarin acts by inhibiting factor XII.
d. If the patient is also taking aspirin, the dose of warfarin should be
reduced.
e. Protamine sulphate is the antidote to warfarin.
86.A mother mentions that her 8-year-old boy gets recurrent attacks of
hemarthrosis following a minimal trauma. Investigations revealed normal
platelet count and prothrombin time, but the partial thromboplastin time is
prolonged. Which of the following statements regarding this clinical condition is
TRUE?
a. There is an underlying liver problem.
b. The sisters of this boy are usually having the same problem.
c. The Loy has had repeated episodes of epistaxis.
d. There is no family history in this condition.
e. Transfusion of factor VIII concentrate is helpful.
87.A 40-year-old male patient is complaining of persistent headache, recurrent
attacks of palpitation, sweating and visual disturbances. Blood pressure of the
patient is 180/110 mmHg. Which of the following is MOST accurate in the
diagnosis of the problem?
a. 24 hour urinary catecholamines.
b. 24 hour urinary VMA.
c. CT scan of the abdomen.
d. Plasma aldosterone level.
e. Plasma metanephrine level.
88.A 30-year-old pregnant female patient (First trimester) developed palpitation,
tremors, excessive sweating, nervousness and failure to gain weight. Which of
the following investigations is the MOST accurate to diagnose her condition?
a. Free thyroxine level.
b. Level of thyroid peroxidase enzyme.
c. Technetium scan.
d. Thyroglobulin level.
e. Total thyroxine level.
P a g e 22 | 25
ElEMAM’s SurgiNotes 2022
89.What is the hormone that acts on the intestines to increase calcium absorption?
a. Calcitonin.
b. Corticotrophin releasing factor (CRF).
c. Pancreatic polypeptide.
d. Parathormone.
e. Thyroxine.
90.Regarding the anatomy of the thyroid gland, which of the following is TRUE?
a. In about 80% of persons, the recurrent laryngeal nerve traverses anterior
to the inferior thyroid artery.
b. The recurrent laryngeal nerve has an oblique course around the
subclavian artery on the left side.
c. The superior laryngeal nerve provides both sensory and motor function
to the larynx.
d. The thyroid gland is innervated only by parasympathetic fibers from the
vagus nerve.
e. Unilateral recurrent laryngeal nerve injury usually results in airway
compromise that necessitates tracheotomy.
91.Which of the following is the MOST potent stimulus for aldosterone secretion?
a. ACTH.
b. Antidiuretic hormone.
c. Hyperkalaemia.
d. Hypernatraemia.
e. The rennin angiotensin system.
95.A 50-year-old woman presents with lethargy, weight gain, cold intolerance and
loss of interest for the past six months. Which is the MOST appropriate initial
investigation?
a. Erythrocyte sedimentation rate estimation.
b. Radioactive isotope scan of thyroid.
c. Thyroid antibodies screen.
d. Thyroid stimulating hormone estimation.
e. Ultrasound scan of thyroid gland.
96.A 48-year-old woman with breast cancer diagnosed five years ago is admitted
with a two week history of back pain, nausea, fatigue and constipation. Her
serum calcium is 14 mg/dl. Which of the following is the CORRECT first line
treatment for this patient?
a. Bisphosphonate infusion.
b. Calcitonin infusion.
c. Dextrose/insulin infusion.
d. Forced diuresis with furosemide.
e. Intravenous fluids.
P a g e 24 | 25
ElEMAM’s SurgiNotes 2022
97.Which of the following which is the MOST effective way in preventing surgical
infection?
a. Antibiotic prophylaxis.
b. Bowel preparation.
c. Drains and irrigation.
d. Skin preparation.
e. Surgical technique.
98.Which of the following is the MOST likely cause of pyrexia occurring 48 hours
after an abdominal operation?
a. Chest infection.
b. DVT.
c. Leaking intestinal anastomosis.
d. Pulmonary embolism.
e. Wound infection.
6. Which of the following is NOT present in ulnar nerve injury at the wrist?
a. Atrophy of the interossei.
b. Inability to adduct the thumb.
c. Loss of sensation over the medial 1/3 of the dorsum of the hand.
d. Partial claw hand.
e. Inability to abduct the fingers.
10.All the following regarding the femoral canal are CORRECT EXPECT:
a. The femoral sheath invests the femoral vessels and the femoral branch of
the genitofemoral nerve.
b. The femoral nerve courses down lateral to the femoral artery outside the
femoral sheath.
c. The femoral hernia descends down within the femoral sheath.
d. The femoral canal is a fatty cushion between the femoral artery and vein.
e. The pectineus muscle lies behind the femoral sheath.
14.Which of the following nerves is NOT derived from the cervical plexus?
a. Auriculotemporal nerve.
b. Great auricular nerve.
c. Transverse cervical nerve.
d. Supraclavicular nerves.
e. Phrenic nerve.
15.Which of the following is FALSE regarding the flexor retinaculum at the wrist?
a. Attached to the scaphoid and trapezium laterally and to the pisiform and
hamate medially.
b. Gives attachments to the thenar and hypothenar eminences muscles.
c. The median nerve enters the palm beneath its midpoint.
d. Ulnar nerve and artery run deep to it medially.
e. Gives an insertion to the palmaris longus muscle.
16.Regarding the inguinal canal all the following statements are TRUE EXCEPT:
a. The internal ring lies midway between the symphysis pubis and anterior
superior iliac spine.
b. The internal ring lies lateral to the inferior epigastric vessels.
c. The external oblique aponeurosis forms the anterior boundary.
d. The inguinal ligament forms the inferior boundary.
e. The conjoint tendon forms the lateral part of the posterior wall.
P a g e 4 | 25
ElEMAM’s SurgiNotes 2022
17.Taste from the posterior one third of tongue is provided by which of the
following nerves?
a. Glossopharyngeal.
b. Facial.
c. Lingual.
d. Vagus.
e. Hypoglossal.
18.All the following are branches of the internal iliac artery EXCEPT:
a. Superior gluteal artery.
b. Inferior vesical artery.
c. Superior rectal artery.
d. Inferior rectal artery.
e. Uterine artery.
19.Which of the following structures does NOT pass through the aortic opening of
the diaphragm?
a. Azygos vein.
b. Thoracic duct.
c. The right phrenic nerve.
d. The aorta.
e. Lymph vessels from the thorax.
23.Which of the following does NOT pass in the adductor (Hunter's) canal?
a. Femoral artery.
b. Saphenous nerve.
c. Nerve to vastus medialis.
d. Obturator nerve.
e. Femoral vein.
24.Which of the following is NOT TRUE about the anatomy of the ureter?
a. It crosses the bifurcation of the common iliac artery.
b. It passes superficial to the uterine artery.
c. The ductus deferens crosses superficial to the ureter near its termination.
d. It receives blood supply from the renal, aortic and vesical arteries.
e. It is supplied by sympathetic autonomic nerves T11-L1.
P a g e 6 | 25
ElEMAM’s SurgiNotes 2022
26.Which of the following is NOT present in injury of the common peroneal nerve?
a. Loss of eversion of the foot.
b. Loss of extension of the foot.
c. Loss of sensations at the sides of the third toe.
d. Loss of sensations at the lateral aspect of the leg.
e. Loss of planter flexion of the foot.
29.Which of the following tests will be affected after intake of non-steroidal anti-
inflammatory drugs?
a. Coagulation time.
b. PTT.
c. INR.
d. Bleeding time.
e. Thrombin time.
35.The MAJOR source of protein for multi-trauma patients during the catabolic
phase is:
a. Plasma protein.
b. Fat.
c. Liver.
d. Skeletal muscle.
e. Kidney.
36.Which of the following cell types is essential for normal wound healing?
a. Leukocytes.
b. Monocytes.
c. Platelets.
d. Erythrocytes.
e. Lymphocytes.
P a g e 9 | 25
ElEMAM’s SurgiNotes 2022
38.Low molecular weight heparin produces its effects due to inhibition of which
factor?
a. IIa.
b. IXa.
c. Xa.
d. XI.
e. XII.
44.Which of the following is the BEST index for the severity of shock?
a. Hb%.
b. Creatinine level.
c. Lactate level.
d. O2 saturation of arterial blood.
e. Plasma sodium concentration.
P a g e 11 | 25
ElEMAM’s SurgiNotes 2022
46.Which of the following does NOT cause increased gastrin release from the
pyloric antrum?
a. Antral distension.
b. Vagal stimulation
c. Products of protein digestion.
d. Antral acidity.
e. Antral alkalinity.
51.Which of the following is NOT TRUE about the physiology of the thyroid gland?
a. Thyroxine is mainly bound to thyroid binding globulin.
b. T3 is 3-4 times more active than T4.
c. Cells of thyroid follicles have prominent endoplasmic reticulum.
d. The half-life of thyroxine is about four days.
e. Thiouracil blocks the binding of iodine to tyrosine.
52.The MOST important difference between interstitial fluid and plasma is the:
a. Osmolarity.
b. Potassium concentration.
c. Sodium concentration.
d. Concentration of proteins.
e. Chloride concentration.
P a g e 13 | 25
ElEMAM’s SurgiNotes 2022
61.The FIRST important event in hemostasis following severe tissue injury is:
a. Blood coagulation.
b. Formation of a platelet plug.
c. Vascular spasm.
d. Formation of thromboplastin.
e. Formation of prothrombin activator.
66.You were obliged to operate for a strangulated inguinal hernia in a patient who
was receiving 150 mg aspirin daily. During surgery there was excessive bleeding.
What would you advise?
a. Fresh blood.
b. Fresh frozen plasma.
c. Desmopressin.
d. IV vitamin K.
e. Fresh platelets.
78.Which of the following is the MOST likely cause of pyrexia occurring 48 hours
after an abdominal operation?
a. Wound infection.
b. DVT.
c. Chest infection.
d. Pulmonary embolism.
e. Leaking intestinal anastomosis.
81.The organism that is FAMOUS for production of the enzyme penicillinase and
resists penicillin therapy is:
a. Staphylococcus aureus.
b. Haemolytic streptococci.
c. Klebsiella.
d. Pseudomonas aeruginosa.
e. Bacteroides.
92.Arterial blood gas analysis revealed a pH 7.6, P(O2) 85 mmHg, P(CO2) 46 mmHg
and HCO3- 44 mmol/L. This denotes:
a. Metabolic acidosis.
b. Respiratory alkalosis.
c. Respiratory acidosis.
d. Metabolic alkalosis.
e. Combined respiratory and metabolic alkalosis.
P a g e 23 | 25
ElEMAM’s SurgiNotes 2022
98.A 20-year-old woman has the following arterial blood results: pH= 7.49, PCO2=
47 mmHg, HCO3-= 35 mmol/L, oxygen saturation= 98% on air. Her plasma
potassium concentration is 2.5 mmol. The physiologic status can BEST be
described as which of the following?
a. There is a respiratory alkalosis.
b. There is a metabolic alkalosis.
c. Her urine is likely to be acidic.
d. Pulmonary embolism is a likely diagnosis.
e. There is respiratory acidosis.
ELEMAM’s SurgiNotes
ElEMAM’s SurgiNotes 2022
2. Explain how can a young fit person lose about 1.5 liters of blood, yet the vital
signs are stable. (14 marks)
4. A 65-year-old fit male patient had a colectomy operation for colonic cancer. The
patient was doing fine in the postoperative period, but on the 7th postoperative
day the patient collapsed while going to the toilet. The pulse was 110/min and
BP was 90/60 mm Hg. The respiratory rate was 24/minute.
a. What is the most likely diagnosis? (1 mark)
b. Mention one differential diagnosis. (1 mark)
c. What investigations you would like to do. (4 marks)
d. What is the treatment? (8 marks)
5.
A. What is the definition of surgical site infection (SSI)? (2 marks)
B. Classify wounds regarding their liability to SSI. (4 marks)
C. Discuss the treatment of necrotizing fasciitis. (6 marks)
Page 1|2
ElEMAM’s SurgiNotes 2022
8.
A. A 50-year-old female patient is diagnosed as having right
pheochromocytoma. The pulse is 120/min and BP is 190/110 mm Hg. You
are planning to operate on the patient.
a. How are you going to prepare this patient for the operation?
(2 marks)
b. Explain 3 complications specific to this operation which may occur
during the operation or in the postoperative period. Discuss how are
going to treat these complications. (5 marks)
B. A 55-year-old female patient has chronic renal failure and she is on
regular renal dialysis. For the last year the patient has been complaining
of generalized bony aches and pruritis. Investigation revealed blood urea
of 50 mg/dl and creatinine of 2 mg/dl, serum calcium 10 mg/dl, serum
phosphates 7 mg/dl, and a high parathormone level.
a. What is the most likely diagnosis? (1 mark)
b. Explain the management. (4 marks)
Page 2|2
ElEMAM’s SurgiNotes 2022
Page 1|3
ElEMAM’s SurgiNotes 2022
2. A 20-year-old male was subjected to a car accident which led to fractures of his
right femur and tibia. Exam revel a conscious patient with a pulse of 80/min and
BP 120/80 mmHg. Explain how the patient had normal vital signs in spite of his
major injuries. (14 marks)
4.
A. Mention 5 indications for liver and for renal transplantation. (5 marks)
B. Mention 3 immunosuppressive drugs and enumerate their side effects.
(7 marks)
5.
A. Discuss the clinical picture and treatment of incompatible blood
transfusion. (6 marks)
B. What is meant by massive blood transfusion? (1 marks)
C. What are the complications of massive blood transfusion? (5 marks)
6.
A. For each of the following drugs explain the mode of action and the
antidote in case of overdosage:
a. Heparin. (2 marks)
b. Warfarin. (2 marks)
c. Streptokinase (2 marks)
d. Aspirin. (2 marks)
B. What is meant by consumption coagulopathy? Mention 5 etiologies for
the problem. (6 marks)
Page 2|3
ElEMAM’s SurgiNotes 2022
7.
A. Mention 5 indications and 5 contraindications for total parenteral
nutrition (TPN). (5 marks)
B. What is catheter related fever and what is the treatment? (7 marks)
Page 3|3
ElEMAM’s SurgiNotes 2022
2.
A. Explain the factors which affect wound healing. (10 marks)
3.
A. Discuss the factors which affect calcium metabolism. (8 marks)
B. Mention the laboratory findings in primary hyperparathyroidism.
(3 marks)
C. Mention 6 causes for Hypercalcaemia. (3 marks)
6.
A. What is the definition of surgical site infection (SSI)? (2 marks)
B. Classily wounds regarding to their liability to SSI. (6 marks)
C. You are planning to perform colectomy for colonic cancer, explain the
procedures you are going to do to minimize SSI? (4 marks)
8.
A. What is meant by screening programmes for cancer? (2 marks)
B. What are the pre-requisites of a good screening programme? (4 marks)
C. Mention 4 problems which are suitable for screening programmes.
(3 marks)
D. What is the definition of the following terms:
a. Metaplasia. (1 mark)
b. Dysplasia. (1 mark)
c. Carcinoma in situ. (1 mark)
Page 2|2
ElEMAM’s SurgiNotes 2022
2.
A. Discuss the factors which affect wound healing. (8 marks)
B. Classify Suture materials. Mention 2 types for each one of your
classification. (6 marks)
3.
A. Mention 3 causes for each of the following: (8 marks)
a. Respiratory acidosis.
b. Metabolic acidosis.
c. Respiratory alkalosis.
d. Metabolic alkalosis.
B. In a table form mention the arterial blood gases (PH, PO2, PCO2 & HCO3-)
of each one of them. (6 marks)
8. A 50- years old diabetic male patient had a scratch of the left thigh. Over the
next 2-dayes the patient started to complain of fever. Pain and swelling of the
thigh as shown in the figure. Pulse was 120/min and temperature 38.5 °C.
a. What is the most likely diagnosis? (1 mark)
b. Mention 3 valuable investigations. (2 marks)
c. Discuss the treatment. (9 marks)
Page 2|2
ElEMAM’s SurgiNotes 2022
Page 1|2
ElEMAM’s SurgiNotes 2022
9. The following are clinical scenarios for 5 clinical cases. Read the scenarios the
answer in your answer sheet. (3 marks each)
a. A 20-year old restrained driver with tachycardia, hypotension and a
……abdomen after collision with another car.
b. A 70-year old male with urine retention due to prostate hyperplasia
undergoes a difficult catheterization. He develops fever, chills, loin pain
and hypertension.
c. A 17-year old male victim of a motor vehicle crash has bradycardia,
……tension and complains that he cannot move or feel his lower limbs.
d. A 65-year old male is brought to the ICU complaining of chest pain and
dyspnoea. He is diaphoretic, hypotensive and has crepitations in the ……of
both lungs.
e. A hydatid cyst of the liver ruptures during surgery. The patient becomes
hypotensive.
Page 2|2
ElEMAM’s SurgiNotes 2022
2.
A. Discuss the stages of wound healing. (4 marks)
B. What are the factors which affect wound healing? (8 marks)
3.
A. Mention 5 cases for metabolic acidosis. (5 marks)
B. Write an arterial blood gas report indicative of metabolic acidosis.
(2 marks)
C. What is the treatment? (5 marks)
4. Explain how a young fit man may lose up to 1.5 L of blood and yet his vital signs
are stable. (14 marks)
5.
A. Mention 5 causes for disseminated intravascular coagulation. (5 marks)
B. Mention the results of haemostatic tests. (3 marks)
C. What is the treatment? (4 marks)
6.
A. Mention 5 indications for total parenteral nutrition (TPN). (5 marks)
B. Discuss the complications of TPN. (7 marks)
7.
A. What is the definition of surgical site infection (SSI)? (2 marks)
B. Classify wounds regarding their liability to SSI. (6 marks)
C. Discuss the clinical picture and treatment of necrotizing fasciitis. (6 marks)
8. Discuss the clinical picture, investigations and treatment of pulmonary
embolism. (12 marks)
Page 1|1
ElEMAM’s SurgiNotes 2022
2. Discuss the relations, blood supply and lymphatic drainage of the right kidney.
(12 marks)
3. Explain how a young fit man may lose 1.5 L of blood, yet he is vitally stable.
(14 marks)
4.
A. Discuss the stages of wound healing. (6 marks)
B. Discuss the factors which affect wound healing. (8 marks)
ELEMAM’s SurgiNotes
ElEMAM’s SurgiNotes 2022
8. Which of the following is NOT present in ulnar nerve injury at the wrist?
a. Atrophy of the interossei.
b. Inability to adduct the thumb.
c. Loss of sensation over the medial 1/3 of the dorsum of the hand.
d. Partial claw hand.
e. Inability to abduct the fingers.
11.Which of the following muscles does NOT belong to the rotator cuff?
a. Supraspinatus.
b. Subscapularis.
c. Teres major.
d. Teres minor.
e. Infraspinatus.
12.Which of the following is NOT present following injury of the radial nerve in the
arm?
a. Wrist drop.
b. Inability to extend the metacarpophalangeal joints.
c. Sensor loss at the radial side of the dorsum of the hand.
d. Inability to extend the forearm.
e. Wasting of muscles at the back of forearm.
15.Which of the following is NOT TRUE about the submandibular salivary gland?
a. The duct arises from the deep part of the gland.
b. The deep part lies between the mylohyoid and the digastric muscle.
c. It is related to the hypoglossal nerve.
d. The mandibular branch of the facial nerve may be injured during
submandibular sialoadenectomy.
e. It is more frequently affected by calculi than the parotid gland.
P a g e 3 | 22
ElEMAM’s SurgiNotes 2022
18.All the following regarding the anatomy of the salivary glands are CORRECT
EXCEPT:
a. The parotid gland is mainly serous.
b. The submandibular gland is mainly mucous.
c. The minor salivary glands are found in all the oral mucosa including the
hard palate.
d. The secretory fibers reach the parotid through the auriculotemporal
nerve.
e. The submandibular duct courses anteriorly below the lingual nerve.
19.All the following regarding the femoral canal are CORRECT EXPECT:
a. The femoral sheath invests the femoral vessels and the femoral branch of
the genitofemoral nerve.
b. The femoral nerve courses down lateral to the femoral artery outside the
femoral sheath.
c. The femoral hernia descends down within the femoral sheath.
d. The femoral canal is a fatty cushion between the femoral artery and vein.
e. The pectineus muscle lies behind the femoral sheath.
P a g e 4 | 22
ElEMAM’s SurgiNotes 2022
P a g e 5 | 22
ElEMAM’s SurgiNotes 2022
27.One of the following muscles is NOT involved in full abduction of the upper limb:
a. Deltoid.
b. Infraspinatus.
c. Serratus anterior.
d. Teres major.
e. Trapezius.
28.The deep perineal pouch contains all the following structures EXCEPT:
a. The membranous urethra.
b. Sphincter urethra.
c. Bulbourethral glands.
d. Internal pudendal vessels.
e. Superficial transverse perineal muscles.
33.All the following statements about the femoral triangle are CORRECT EXCEPT:
a. The lateral border is formed by the medial border of the sartorius.
b. The medial border is formed by the lateral border of adductor longus.
c. The floor is formed by iliacus, psoas and pectineus and adductor longus.
d. The profunda femoris vessels are among the contents.
e. The femoral branch of the genitofemoral nerve lies in the roof of the
triangle.
34.Which nerve is responsible for causing pain in the ear during acute tonsillitis?
a. Facial.
b. Lingual.
c. Glossopharyngeal.
d. Lesser palatine.
e. Hypoglossal.
36.Which of the following is NOT TRUE about the psoas major muscle?
a. It originates in part from the last thoracic vertebra.
b. It is supplied from the femoral nerve.
c. It is inserted into the lesser trochanter.
d. It is a flexor to the hip joint.
e. The genitofemoral nerve is anterior to it.
37.Which of the following nerves is NOT derived from the cervical plexus?
a. Auriculotemporal nerve.
b. Greater auricular nerve.
c. Transverse cervical nerve.
d. Supraclavicular nerves.
e. Phrenic nerve.
38.All the following regarding the submental triangle are CORRECT EXCEPT:
a. It is bound by the anterior bellies of the digastric muscle and the hyoid
bone.
b. The mylohyoid muscle constitutes its floor.
c. Lymph nodes constitutes its major anatomical content.
d. The sublingual salivary glands are amongst its contents.
e. Thyroglossal duct cyst can be found at its base.
P a g e 8 | 22
ElEMAM’s SurgiNotes 2022
42.Which of the following is FALSE regarding the flexor retinaculum at the wrist?
a. Attached to the scaphoid and trapezium laterally and to the pisiform and
hamate medially.
b. Gives attachments to the thenar and hypothenar eminences muscles.
c. The median nerve enters the palm beneath its midpoint.
d. Ulnar nerve and artery run deep to it medially.
e. Gives an insertion to the palmaris longus muscle.
43.Regarding the inguinal canal all the following statements are TRUE EXCEPT:
a. The internal ring lies midway between the symphysis pubis and anterior
superior iliac spine.
b. The internal ring lies lateral to the inferior epigastric vessels.
c. The external oblique aponeurosis forms the anterior boundary.
d. The inguinal ligament forms the inferior boundary.
e. The conjoint tendon forms the lateral part of the posterior wall.
44.Taste from the posterior one third of tongue is provided by which of the
following nerves?
a. Glossopharyngeal.
b. Facial.
c. Lingual.
d. Vagus.
e. Hypoglossal.
45.Pain referred to the right side of the neck and extending laterally from the right
clavicle to the tip of the right shoulder is MOST likely due to involvement of:
a. Cervical cardiac nerves.
b. Posterior vagal trunk.
c. Right intercostal nerves.
d. Right phrenic nerve.
e. Right recurrent laryngeal nerve.
P a g e 9 | 22
ElEMAM’s SurgiNotes 2022
47.A complete division of the right oculomotor nerve (III) would result in all of the
following signs EXCEPT:
a. Ptosis.
b. Diplopia.
c. Convergent squint.
d. Dilated pupil on the right side.
e. Loss of a consensual papillary reflex when the left eye is examined.
48.All of the following muscles are involved in abduction of the hip joint EXCEPT:
a. Gluteus medius.
b. Gluteus minimus.
c. Sartorius.
d. Quadratus femoris.
e. Tensor fasciae latae.
49.All of the following structures would drain through the thoracic duct EXCEPT:
a. Left arm and thorax.
b. Left face and neck.
c. Left side of abdomen.
d. Right face and neck.
e. Right side of abdomen.
50.Which of the following nerves provides the afferent limb of the sneezing reflex?
a. Ophthalmic division of the trigeminal nerve.
b. Maxillary division of the trigeminal nerve.
c. Mandibular division of the trigeminal nerve.
d. Glossopharyngeal nerve.
e. Vagus nerve.
P a g e 10 | 22
ElEMAM’s SurgiNotes 2022
52.All the following are branches of the internal iliac artery EXCEPT:
a. Superior gluteal artery.
b. Inferior vesical artery.
c. Superior rectal artery.
d. Inferior rectal artery.
e. Uterine artery.
53.Which of the following structures does NOT pass through the aortic opening of
the diaphragm?
a. Azygos vein.
b. Thoracic duct.
c. The right phrenic nerve.
d. The aorta.
e. Lymph vessels from the thorax.
P a g e 11 | 22
ElEMAM’s SurgiNotes 2022
70.Which of the following will be present following injury of the long thoracic
nerve?
a. Depression of the shoulder.
b. Inability to extend the shoulder.
c. Winging of the scapula.
d. Inability to abduct the shoulder.
e. Loss of sensation at the medial side of the axilla.
P a g e 14 | 22
ElEMAM’s SurgiNotes 2022
76.Which of the following muscle does NOT belong to the rotator cuff?
a. Subscapularis.
b. Teres minor.
c. Supraspinatus.
d. Infraspinatus.
e. Teres major.
77.Which of the following structures passes into the quadrangular space of the
shoulder?
a. Anterior circumflex humeral artery.
b. Radial nerve.
c. Posterior circumflex humeral artery.
d. Circumflex scapular artery.
e. Thoracodorsal nerve.
80.Ligation of which of the following arteries is MOST likely to affect the blood
supply of the pancreas?
a. Inferior phrenic.
b. Superior mesenteric.
c. Left gastroepiploic.
d. Inferior mesenteric.
e. Right colic.
P a g e 16 | 22
ElEMAM’s SurgiNotes 2022
81.Which of the following does NOT pass in the adductor (Hunter's) canal?
a. Femoral artery.
b. Saphenous nerve.
c. Nerve to vastus medialis.
d. Obturator nerve.
e. Femoral vein.
82.Which of the following nerves emerges at the medial margin of the psoas major
muscle?
a. Ilioinguinal.
b. Femoral.
c. Genitofemoral.
d. Obturator.
e. Iliohypogastric.
85.Which of the following vessels does NOT participate in the blood supply of the
hip joint?
a. Medical circumflex femoral.
b. Superior gluteal.
c. Lateral circumflex femoral.
d. Obturator.
e. Pudendal.
P a g e 17 | 22
ElEMAM’s SurgiNotes 2022
89.If the inferior mesenteric artery is occluded, which artery will supply the
descending colon?
a. Superior rectal.
b. Middle rectal.
c. Marginal artery.
d. Left gastroepiploic.
e. Ileocolic.
90.Ligation of which of the following arteries is MOST likely to affect the blood
supply of the pancreas?
a. Inferior phrenic.
b. Superior mesenteric.
c. Left gastroepiploic.
d. Inferior mesenteric.
e. Right colic.
P a g e 18 | 22
ElEMAM’s SurgiNotes 2022
94.Which of the following is NOT TRUE regarding the pterygoid venous plexus?
a. It is situated between the temporalis and lateral pterygoid muscles.
b. It communicates with the anterior facial vein.
c. It drains into the external jugular vein.
d. It communicates with the cavernous sinus.
e. It receives the middle meningeal vein.
96.The MAIN blood supply of the upper end of the esophagus is:
a. Inferior thyroid artery.
b. Transverse cervical artery.
c. Suprascapular artery.
d. Aorta.
e. Left bronchial artery.
98.Which of the following is NOT TRUE about the anatomy of the ureter?
a. It crosses the bifurcation of the common iliac artery.
b. It passes superficial to the uterine artery.
c. The ductus deferens crosses superficial to the ureter near its termination.
d. It receives blood supply from the renal, aortic and vesical arteries.
e. It is supplied by sympathetic autonomic nerves T11-L1.
100. Which of the following nerves is responsible for motor innervation of the
constrictor muscles?
a. The vagus nerve.
b. The glossopharyngeal nerve.
c. The accessory nerve.
d. The sympathetic trunk.
e. The hypoglossal nerve.
P a g e 20 | 22
ElEMAM’s SurgiNotes 2022
101. Which of the following muscles is NOT supplied by the oculomotor nerve?
a. Levator palpebrae superioris.
b. Superior oblique.
c. Medial rectus.
d. Inferior oblique.
e. Superior rectus.
102. Which of the following is NOT TRUE as regard the pudendal nerve?
a. It arises from S2,3,4.
b. The leaves the pelvis through the greater sciatic foreman.
c. It runs along alcohol's canal.
d. It is a purely sensory nerve.
e. It gives off the inferior rectal nerve.
104. Which of the following is NOT TRUE regarding the cavernous sinus?
a. The right and left sinuses are connected together.
b. It drains into the inferior petrosal sinus.
c. It receives blood from the superior and inferior ophthalmic veins.
d. Infection of the central area of the face may cause cavernous sinus
thrombosis.
e. It contains the mandibular nerve.
105. All the following are branches of the maxillary artery EXCEPT:
a. Middle meningeal artery.
b. Accessory meningeal artery.
c. Deep temporal arteries.
d. Inferior alveolar artery.
e. Ascending palatine.
P a g e 21 | 22
ElEMAM’s SurgiNotes 2022
106. Which of the following muscles is NOT supplied by the mandibular nerve?
a. Lateral pterygoid.
b. Temporalis.
c. Posterior belly of digastric.
d. Stylohyoid.
e. Mylohyoid.
107. Which of the following muscles is NOT supplied by the vagus nerve?
a. Laryngeal muscles.
b. Pharyngeal constrictors.
c. Palatoglossus.
d. Genioglossus.
e. Palatopharyngeus.
108. Injury of the hypoglossal nerve causes all the following EXCEPT:
a. Atrophy of the same side of the tongue.
b. Deviation of the tongue to the same side of the lesion.
c. Paralysis of all the muscles of the tongue.
d. ……
e. ……
P a g e 22 | 22
ElEMAM’s SurgiNotes 2022
10.Vancomycin:
a. 90% of vancomycin is excreted by glomerular filtration.
b. Inhibits protein synthesis in bacteria.
c. Is bactericidal against gram negative bacilli.
d. Is well absorbed from the GIT.
e. One adverse reaction to infusions of vancomycin is the “Blue man”
syndrome.
Page 2|8
ElEMAM’s SurgiNotes 2022
13.Chloramphenicol:
a. Does not penetrate the blood brain barrier.
b. Must be administered parenterally.
c. Can be safely used in premature infants.
d. Can cause depression of bone marrow function.
e. Can cause discoloration of developing teeth when given to children.
14.Spironolactone:
a. Has a steroid structure.
b. Is a partial agonist.
c. Promotes sodium retention.
d. Increases potassium loss.
e. Is a loop diuretic.
16.All of the following antibiotics bind to the 50S subunit of the ribosome thereby
inhibiting protein synthesis EXCEPT:
a. Chloramphenicol.
b. Erythromycin.
c. Linezolid.
d. Doxycycline.
e. Clindamycin.
17.Pharmacokinetics of doxycycline:
a. 20% bound by serum proteins.
b. 60-70% absorption after oral administration.
c. Absorption is impaired by divalent cations, Al3+ and antacids.
d. Widely distributed especially into the CSF.
e. Is eliminated via renal mechanisms.
21.Regarding fluoroquinolones:
a. Ciprofloxacin is ineffective in the treatment of gonococcus.
b. Norfloxacin and Ciprofloxacin are predominantly faecally excreted.
c. Norfloxacin and Ciprofloxacin have long half-lives (12 hours).
d. They have poor oral bioavailability.
e. May damage growing cartilage in children less than 18 years of age.
22.Vancomycin:
a. Is never orally administered as it is poorly absorbed from the GIT.
b. Binds to the 30S unit on the ribosome and inhibits protein synthesis.
c. 60% of vancomycin is excreted by glomerular filtration.
d. Parenteral vancomycin is commonly used for treatment of infections
caused by methicillin susceptible staphylococci.
e. Adverse reactions to vancomycin are encountered in about 10% of
patients.
23.Regarding the “Azole” group of antifungals:
a. Fluconazole has low water solubility.
b. Ketoconazole may be given IV/PO.
c. Itraconazole undergoes renal elimination.
d. Clotrimazole is the treatment of choice for systemic candidiasis given
orally.
e. They work by reduction of ergosterol synthesis by inhibition of fungal
cytochrome P450 enzymes.
24.The fluoroquinolones:
a. May be administered to patients with severe campylobacter infection.
b. Work by inhibiting dihydrofolate reductase.
c. Have little effect against gram positive organisms.
d. Are heavily metabolized in the liver.
e. Are safe to give to breast feeding mothers.
25.Clindamycin:
a. Inhibits bacterial cell wall synthesis.
b. Is often used for prophylaxis of endocarditis in patients with valvular
disease who are undergoing dental procedures.
c. Penetrates through BBB into CSF well.
d. Works well against enterococci and gram negative aerobic organisms.
e. Is 10% protein bound.
Page 5|8
ElEMAM’s SurgiNotes 2022
27.The cephalosporin with the HIGHEST activity against gram positive cocci is:
a. Cefaclor.
b. Cephalothin.
c. Cefuroxime.
d. Cefepime.
e. Cefotaxime.
29.Rifampicin:
a. Inhibits hepatic microsomal enzymes.
b. Inhibits DNA synthesis.
c. Is bactericidal for mycobacteria.
d. Is not appreciably protein bound.
e. Is predominantly excreted unchanged in the urine.
35.Ciprofloxacin:
a. Is a defluorinated analogue of nalidixic acid.
b. Inhibits topoisomerases 2 and 3.
c. Has no gram positive cover.
d. Has bioavailability of 30%.
e. May cause an arthropathy.
Page 7|8
ElEMAM’s SurgiNotes 2022
36.Flucloxacillin:
a. Is ineffective against streptococci.
b. Is active against enterococci and anaerobes.
c. Blocks transpeptidation and inhibits peptidoglycan synthesis.
d. Is poorly absorbed orally.
e. Has excellent penetration into CNS and prostate.
37.Aminoglycosides:
a. Have a β lactam ring.
b. Can produce neuromuscular blockade.
c. Are DNA gyrase inhibitors.
d. Normally reach high CSF concentrations.
e. Have good oral absorption but high first pass metabolism.
39.Regarding antivirals:
a. Delvindine is a nucleoside reverse transcriptase inhibitor (NRTI).
b. Zidovudine (AZT) is a non-nucleoside reverse transcriptase inhibitor
(NNRTI).
c. NRTIs activate HIV-1 reverse transcriptase.
d. Abacavir is a protease inhibitor.
e. NRTIs require intracytoplasmic activation to the triphosphate form.
8. In regard to pH:
a. pH of a solution is the log to base 10 of the reciprocal hydrogen ion
concentration.
b. Is the negative log of the concentration of hydrogen ions.
c. For each pH unit less than 7, the concentration of hydrogen ion is
increased 10 fold.
d. A pH of 7 is equal to a hydrogen ion concentration of 10-7 mmol/L.
e. All of the above are true.
11.Regarding basic physiological measures all of the following are TRUE EXCEPT:
a. Osmolarity is the number of osmoles/liter of solution.
b. pH is the log to base 10 of the reciprocal of hydrogen ion concentration.
c. Carbon has a molecular mass of 12 Dalton.
d. Osmolarity is measured by freezing point depression.
e. One equivalent of sodium is 23 g/L.
13.Regarding the comparison of ECF with CSF, all of the following are TRUE EXCEPT:
a. CSF has less protein.
b. CSF has lower osmolality.
c. CSF has lower pH.
d. CSF has more bicarbonate.
e. CSF has lower specific gravity.